You are on page 1of 151

Precalculus

SENIOR
HIGH
SCHOOL

Module

1
Quarter 3

The Limit of a Function


EXPECTATION

Lesson: The Limit of a Function

Learning Objectives:

At the end of the learning episode, you are expected to:


1. illustrate the limit of a function using a table of values and the graph
of the function; and
2. distinguish between lim 𝑓(𝑥) and 𝑓(𝑐).
𝑥→𝑐

PRETEST

A. For each item, complete the table of values, and use the results to
estimate the value of the limit.
1. lim (𝑥 2 + 2𝑥 − 3)
𝑥→4

x 3.9 3.99 3.999 4 4.001 4.01 4.1


f(x) ?
𝑥 2 +𝑥−2
2. lim
𝑥→1 𝑥−1

x 0.9 0.99 0.999 1 1.001 1.01 1.1


f(x) ?

B. Determine if lim 𝑓(𝑥) = 𝑓(𝑐).


𝑥→𝑐
1. 𝑓(𝑥) = 𝑥 + 2; 𝑐 = −1
2. 𝑓(𝑥) = 𝑥 2 − 1; 𝑐 = −1
𝑥 3 −𝑥
3. 𝑓(𝑥) = ;𝑐 = 0
𝑥

C. Evaluate each indicated limit using the given graph.


1. lim 𝑓(𝑥) 2. lim 𝑓(𝑥)
𝑥→2 𝑥→0
3.. lim 𝑓(𝑥) 4. lim 𝑓(𝑥)
𝑥→0 𝑥→1

RECAP

It is important to first recall the important notions about functions. By


a function f with domain D, we mean a way of assigning to each object (usually
a real number) 𝑥 ∈ 𝐷 a unique object (usually a real number), which we denote
𝑓(𝑥). So, a function is always defined by specifying what is assigned to each
number in its domain. The numbers assigned, i.e., the numbers 𝑓(𝑥) where
𝑥 ∈ 𝐷, form what is called the range of the function.

A function is also defined as a set of pairs (𝑥, 𝑦) with the condition that
no two distinct pairs share the same first element. With this notation, we
usually write the “rule” of the assignment as the equation 𝑦 = 𝑓(𝑥). We are,
thus prompted to call x the independent variable as opposed to the
dependent variable y whose value depends on x.

It will also be helpful to us if we can recall the different kinds of


functions, the piecewise function, and how the graph of these functions are
sketched in a Cartesian Plane.

LESSON

𝑥 2 −9
Consider the function f defined by 𝑓(𝑥 ) = . We will investigate the
𝑥−3
function values (i.e., y’s or f(x)’s) as x gets closer and closer to 3. Here we are
not concerned with the value of the function when 𝑥 = 3. In fact, for the given
function f, 𝑓(3) is undefined in which case, it is customary to indicate this
with an open circle when sketching the graph of the function f, at 𝑥 = 3.
If x is in the domain of f, then 𝑥 ≠ 3 so that

𝑥 2 − 9 (𝑥 − 3)(𝑥 + 3)
𝑓 (𝑥 ) = = = 𝑥 + 3.
𝑥−3 𝑥−3
So, we can say that 𝑓 (𝑥 ) = 𝑥 + 3, 𝑥 ≠ 3. The
graph of 𝑓(𝑥) is shown in Figure 1 at the right.

Figure 1
Table 1:

x 0 1 2 2.5 2.9 2.99 2.999 2.9999 2.99999


f(x) 3 4 5 5.5 5.9 5.99 5.999 5.9999 5.99999

Table 2:

x 6 5 4 3.5 3.1 3.01 3.001 3.0001 3.00001


f(x) 9 8 7 6.5 6.1 6.01 6.001 6.0001 6.00001

In Table 1, we let x approach 3 by starting from 0 and moving toward 3


from the left of 3. In Table 2, we let x approach 3 by starting from 6 and
moving toward 3 from the right of 3.

Notice that from Table 1 and 2, as x gets closer and closer to 3, 𝑓(𝑥)
gets closer and closer to 6. The number 6, which 𝑓(𝑥) gets close to when x
gets closer to, but not equal to, 3 is said to be the limit of 𝑓(𝑥) as x approaches
3. In symbols, we write lim 𝑓 (𝑥 ) = 6. We can make 𝑓(𝑥) as close as we like to
𝑥→3
6 by keeping x close enough to 3.

Definition of the Limit of a Function


Let f be a function at every number in some open interval
containing c, except possibly at the number c itself. If the value of f is
arbitrarily close to the number L for all the values of x sufficiently close to
c, then the limit of 𝑓(𝑥) as x approaches c is L. In symbols,

lim 𝑓(𝑥 ) = 𝐿
𝑥→𝑐

Example 1: Evaluate lim 2𝑥 2 using the table of values.


𝑥→−1

Solution: Assign the values of x that are close to 1 and evaluate the function
at those values.
From the left of -1:
x -3 -2 -1.5 -1.1 -1.01 -1.001 -1.0001
𝑓 (𝑥 ) = 2𝑥 2 18 8 4.5 2.42 2.0402 2.004 2.0004

From the right of -1:


x 1 0 -0.5 -0.9 -0.99 -0.999 -0.9999
𝑓 (𝑥 ) = 2𝑥 2 2 0 0.5 1.62 1.9602 1.996 1.9996

By observing the two tables, we will see that


from both directions, as x assumes values closer to -
1, the value of 𝑓(𝑥 ) = 2𝑥 2 becomes closer to 2. Thus,
lim 2𝑥 2 = 2.
𝑥→−1

This is clearly shown in Figure 2.

Figure 2

Example 2: Evaluate the indicated limit using the graph: lim 𝑓(𝑥)
𝑥→0

Figure 3
Solution: The given is an example of a piecewise function, a function that
comes from combining two or more different functions. In fact, the function
is defined by
𝑥 + 2 𝑖𝑓 𝑥 ≠ 0
𝑓 (𝑥 ) = { }
1 𝑖𝑓 𝑥 = 0
Using the definition of limit of a function, the limit of this function is 2
even if it is evident that 𝑓(0) = 1 (See Figure 3). As f approaches 𝑥 = 0 from
both directions, the value of y approaches its “intended” value, which is 2.
Thus,
lim 𝑓(𝑥) = 2.
𝑥→0
Limit and Function Value
The limit of a function as it approaches 𝑥 = 𝑐 is not necessarily equal
to its value at c. Thus, lim 𝑓(𝑥) can assume a value different from 𝑓(𝑐).
𝑥→𝑐

Example 3: Evaluate the following limits: lim 𝑓(𝑥) and lim 𝑔(𝑥)
𝑥→0 𝑥→−1

Figure 4

Solution: The specified limits do not exist. In the first function f, the limit does
not exist because 𝑓(0) is undefined and as 𝑥 → 0, the graph (from the left and
from the right) moves to opposite directions. In the second function g, the limit
does not exist because the function must approach the same value as x
approaches c from both directions. From the graph, the limit of g as 𝑥 → 1
from the left is 2, while the limit of g as 𝑥 → 1 from the right is 3.

Existence of a Limit
The limit of a function as 𝑥 → 𝑐 exists if

• 𝑓(𝑐) is defined; or
• if 𝑓(𝑐) is not defined, then f must approach the same value as x
moves closer to c from both directions
ACTIVITIES

Evaluate the given limits numerically (using table of values) and graphically:

𝑥 2 −4
1. lim
𝑥→2 𝑥−2

|𝑥−2|
2. lim
𝑥→2 𝑥−2

𝑥 𝑖𝑓 𝑥 ≠ 2
3. lim 𝑓(𝑥) when 𝑓 (𝑥 ) = { }
𝑥→2 3 𝑖𝑓 𝑥 = 2

WRAP–UP

To wrap-up, answer the following questions:

1. What is the definition of a limit of a function?

2. How can we get the limit of a function using table of values? How about
graphically?

3. How do you differentiate the limit of a function from a function value?

4. When do we say that the limit of a function exist?

VALUING

The limit of a function at a specified value of x gives us a value to which


it is not possible to go beyond. Similarly, we have our own limitations. We are
restricted to do things beyond our human capacities.
How do you relate the idea of the existence of the limit of a function in
life as a student? as a family member? as part of the community?
POSTTEST

A. Using table of values, determine the limits of the following:


𝑥 2 −1
1. lim 𝑥 2. lim
𝑥→0 𝑥→1 𝑥−1

B. The graph of 𝑓(𝑥 ) is given in Figure 5. Determine the following limits:


1. lim 𝑓(𝑥) 4. lim 𝑓(𝑥)
𝑥→−4 𝑥→3

2. lim 𝑓(𝑥) 5. lim 𝑓(𝑥)


𝑥→−1 𝑥→5

3. lim 𝑓(𝑥)
𝑥→1

Figure 5
C. Determine if lim 𝑓(𝑥) = 𝑓(𝑐).
𝑥→𝑐

1. 𝑓(𝑥) = 𝑥 − 2; 𝑐 = 0

2. 𝑓(𝑥) = 𝑥 2 − 4; 𝑐 = 2
𝑥 2 −1
3. 𝑓(𝑥) = ;𝑐 =1
𝑥−1
KEY TO CORRECTION
Precalculus
SENIOR
HIGH
SCHOOL

Module

2
Illustration of
Quarter 3

Limit Laws
EXPECTATION

Lesson: Illustration of Limit Laws

Learning Objective:

At the end of the learning episode, you are expected to:


1. illustrate the limit laws.

PRETEST

A. Find the given limit.


1. lim 1 2. lim 𝑥
𝑥→−2 𝑥→−2
B. Given that lim 𝑓(𝑥) = 5 and lim 𝑔(𝑥) = 9, find the following limits by applying the
𝑥→𝑐 𝑥→𝑐
limit laws:
𝑓(𝑥)
1. lim[𝑓(𝑥) + 𝑔(𝑥)] 4. lim
𝑥→𝑐 𝑥→𝑐 𝑔(𝑥)

2. lim[𝑓(𝑥) − 𝑔(𝑥)] 5. lim √𝑔(𝑥)


𝑥→𝑐 𝑥→𝑐

3. lim[𝑓(𝑥)𝑔(𝑥)] 6. lim [2 ∗ 𝑓(𝑥)]


𝑥→𝑐 𝑥→𝑐

RECAP

Definition of the Limit of a Function


Let f be a function at every number in some open interval containing c,
except possibly at the number c itself. If the value of f is arbitrarily close to
the number L for all the values of x sufficiently close to c, then the limit of
𝑓(𝑥) as x approaches c is L. In symbols,

lim 𝑓 (𝑥 ) = 𝐿
𝑥→𝑐
Limit and Function Value
The limit of a function as it approaches 𝑥 = 𝑐 is not necessarily equal to
its value at c. Thus, lim 𝑓(𝑥) can assume a value different from 𝑓(𝑐).
𝑥→𝑐

Existence of a Limit
The limit of a function as 𝑥 → 𝑐 exists if

• 𝑓(𝑐) is defined; or
• if 𝑓(𝑐) is not defined, then f must approach the same value as x moves
closer to c from both directions

In the previous module, you have learned that the limit of a function
can be determined in two ways: (1) construct a table of values and study the
behavior of the function values as x approaches its indicated value, or (2)
sketch the graph of the function and study the behavior of the function as it
goes closer to the given value of x from both directions. But these methods
can be time-consuming and tedious. Nonetheless, following certain rules on
limits can help simplify this process.

LESSON

Limit Laws:

In the following statements, c is a constant, and f and g are functions


which may or may not have c in their domains. We also assume that the limits
of f and g both exist as x approaches c and that they are L and M, respectively.
In other words, lim 𝑓(𝑥) = 𝐿 and lim 𝑔(𝑥) = 𝑀.
𝑥→𝑐 𝑥→𝑐

1. Constant Rule. If k is any constant, then lim 𝑘 = 𝑘


𝑥→𝑐

Examples:

lim 4 = 4 lim 4 = 4 lim 4 = 4 lim 4 = 4


𝑥→0 𝑥→−3 𝑥→1/2 𝑥→−5/4

2. Identity Rule. lim 𝑥 = 𝑐


𝑥→𝑐

Examples:
2
lim 𝑥 = 5 lim 𝑥 = −1 lim 𝑥 = lim 𝑥 = 0
𝑥→5 𝑥→−1 𝑥→2/3 3 𝑥→0
3. Constant Multiple Rule. If k is any constant, then

lim [𝑘 ∗ 𝑓 (𝑥 )] = 𝑘 lim 𝑓 (𝑥 ) = 𝑘 ∗ 𝐿
𝑥→𝑐 𝑥→𝑐

Examples: If lim 𝑓(𝑥) = 3, then lim [4 ∗ 𝑓(𝑥 )] = 4 lim 𝑓 (𝑥 ) = 4 ∗ 3 = 12


𝑥→𝑐 𝑥→𝑐 𝑥→𝑐

1 1
If lim 𝑓(𝑥) = , then lim [−2 ∗ 𝑓 (𝑥 )] = −2 lim 𝑓 (𝑥 ) = −2 ∗ = −1
𝑥→𝑐 2 𝑥→𝑐 𝑥→𝑐 2

4. Sum Rule. lim [𝑓(𝑥 ) + 𝑔(𝑥 )] = lim 𝑓 (𝑥 ) + lim 𝑔(𝑥 ) = 𝐿 + 𝑀


𝑥→𝑐 𝑥→𝑐 𝑥→𝑐

Examples: If lim 𝑓(𝑥) = 1 and lim 𝑔(𝑥) = −3, then


𝑥→𝑐 𝑥→𝑐

lim [𝑓 (𝑥 ) + 𝑔(𝑥 )] = lim 𝑓(𝑥 ) + lim 𝑔(𝑥 ) = 1 + (−3) = −2


𝑥→𝑐 𝑥→𝑐 𝑥→𝑐

5. Difference Rule. lim [𝑓 (𝑥 ) − 𝑔(𝑥 )] = lim 𝑓 (𝑥 ) − lim 𝑔(𝑥 ) = 𝐿 − 𝑀


𝑥→𝑐 𝑥→𝑐 𝑥→𝑐

Examples: If lim 𝑓(𝑥) = 1 and lim 𝑔(𝑥) = −3, then


𝑥→𝑐 𝑥→𝑐

lim [𝑓(𝑥 ) − 𝑔(𝑥 )] = lim 𝑓 (𝑥 ) − lim 𝑔(𝑥 ) = 1 − (−3) = 4


𝑥→𝑐 𝑥→𝑐 𝑥→𝑐

6. Product Rule. lim [𝑓 (𝑥 ) ∗ 𝑔(𝑥 )] = lim 𝑓 (𝑥 ) ∗ lim 𝑔(𝑥 ) = 𝐿 ∗ 𝑀


𝑥→𝑐 𝑥→𝑐 𝑥→𝑐

Examples: If lim 𝑓(𝑥) = 1 and lim 𝑔(𝑥) = −3, then


𝑥→𝑐 𝑥→𝑐

lim [𝑓 (𝑥 ) ∗ 𝑔(𝑥 )] = lim 𝑓 (𝑥 ) ∗ lim 𝑔(𝑥 ) = 1 ∗ (−3) = −3


𝑥→𝑐 𝑥→𝑐 𝑥→𝑐

Note:

• The Sum, Difference, and Product Rules may be applied to sums, differences,
and products of more than two functions.
• The Constant Multiple Rule is a special case of the Product Rule. Indeed, in
the Product Rule, if the first function 𝑓(𝑥) is a constant function, the result is
the Constant Multiple Rule.

𝑓(𝑥) lim 𝑓(𝑥) 𝐿


7. Quotient Rule. lim = 𝑥→𝑐 = 𝑀 , 𝑝𝑟𝑜𝑣𝑖𝑑𝑒𝑑 𝑀 ≠ 0.
𝑥→𝑐 𝑔(𝑥) lim 𝑔(𝑥)
𝑥→𝑐

Examples: If lim 𝑓(𝑥) = 1 and lim 𝑔(𝑥) = −3, then


𝑥→𝑐 𝑥→𝑐

lim 𝑓 (𝑥 )
𝑓(𝑥) 𝑥→𝑐 1 1
lim = = =−
𝑥→𝑐 𝑔(𝑥) lim 𝑔(𝑥 ) −3 3
𝑥→𝑐
𝑓(𝑥) lim 𝑓(𝑥) 2
If lim 𝑓(𝑥) = 2 and lim 𝑔(𝑥) = 0, then lim = 𝑥→𝑐 = 0 which not
𝑥→𝑐 𝑥→𝑐 𝑥→𝑐 𝑔(𝑥) lim 𝑔(𝑥)
𝑥→𝑐
defined. Therefore, the limit does not exist.

8. Power Rule. If n is a positive integer, then

lim [𝑓 (𝑥 )]𝑛 = [lim 𝑓 (𝑥 )]𝑛 = 𝐿𝑛


𝑥→𝑐 𝑥→𝑐

Examples: If lim 𝑓(𝑥) = 4, then lim [𝑓 (𝑥 )]2 = [lim 𝑓 (𝑥 )]2 = 42 = 16


𝑥→𝑐 𝑥→𝑐 𝑥→𝑐

9. Root Rule. If n is a positive integer, then

𝑛 𝑛
lim √𝑓(𝑥) = 𝑛√ lim 𝑓(𝑥) = √𝐿
𝑥→𝑐 𝑥→𝑐

3 3
Examples: If lim 𝑓(𝑥) = 8, then lim √𝑓(𝑥) = 3√ lim 𝑓(𝑥) = √8 = 2
𝑥→𝑐 𝑥→𝑐 𝑥→𝑐

If lim 𝑓(𝑥) = −4, then lim √𝑓(𝑥) = √ lim 𝑓(𝑥) = √−4 which is not
𝑥→𝑐 𝑥→𝑐 𝑥→𝑐

a real number. Therefore, the limit does not exist.

ACTIVITIES

A. Evaluate the given limits.

1. lim 0 3. lim 6
𝑥→−1 𝑥→0
2. lim 𝑥 4. lim1 𝑥
𝑥→−1 𝑥→
2
B. Given lim 𝑓(𝑥) = −1 and lim 𝑔(𝑥) = 5, evaluate the following limits:
𝑥→𝑐 𝑥→𝑐
1. lim[3 ∗ 𝑓(𝑥)] 4. lim [𝑓(𝑥)𝑔(𝑥)]
𝑥→𝑐 𝑥→𝑐

𝑓(𝑥)
2. lim[𝑓(𝑥) + 𝑔(𝑥)] 5. lim
𝑥→𝑐 𝑥→𝑐 𝑔(𝑥)

3. lim[𝑓(𝑥) − 𝑔(𝑥)] 6. lim √𝑓(𝑥)


𝑥→𝑐 𝑥→𝑐
WRAP–UP

To wrap-up, illustrate the following laws/rules on limits:

1. Constant Rule 6. Product Rule

2. Identity Rule 7. Quotient Rule

3. Constant Multiple Rule 8. Power Rule

4. Sum/Difference Rule 9. Root Rule

VALUING

In life, we are sometimes taught on different process on how to solve a


specific problem. Some of these problems, if not all, are tedious to solve. Like
in finding the limit of a function that the table of values and sketch of the
graph are used. But, there are some rules or laws that will find the limit of a
function easier and faster that we can relate to our own problems and
challenges in life. At first, we hardly understand them because of their
complex quality, but because of certain rules and laws, solving them will be
easier.
As a student, what are your experiences in life that you realized that
following certain rules or laws made your problem solving easier?
POSTTEST

A. Evaluate the given limits.

2
1. lim 3 3. lim √2
𝑥→0 𝑥→−3
2. lim5 𝑥 4. lim 𝑥
𝑥→ 𝑥→−1
2
1
B. Given lim 𝑓(𝑥) = 4 and lim 𝑔(𝑥) = 4, evaluate the following limits:
𝑥→𝑐 𝑥→𝑐
1. lim[−2 ∗ 𝑓(𝑥)] 2. lim [𝑓(𝑥)𝑔(𝑥)]
𝑥→𝑐 𝑥→𝑐

𝑓(𝑥)
3. lim[𝑓(𝑥) + 𝑔(𝑥)] 5. lim
𝑥→𝑐 𝑥→𝑐 𝑔(𝑥)

4. lim[𝑓(𝑥) − 𝑔(𝑥)] 6. lim √𝑔(𝑥)


𝑥→𝑐 𝑥→𝑐
KEY TO CORRECTION

REFERENCES

BOOK

Canlapan, Raymond B. Basic Calculus. Diwa Learning Systems, Inc., Makati


City. 2017
Cuaresma, Genaro A. et al. 2004. Analytic Geometry and Calculus 1: A
Worktext for Math 26. Los Baños, Laguna: Institute of Mathematical
Sciences and Physics, University of the Philippines.
Department of Education-Bureau of Learning Resources. 2016. Precalculus
Learner's Material.

Leithold, Louis. 1989. College Algebra and Trigonometry. Addison Wesley


Longman Inc., reprinted by Pearson Education Asia Pte. Ltd., 2002.
SENIOR
Basic Calculus HIGH
SCHOOL

Module

3
Quarter 3
Limits of Polynomial,
Rational, & Radical
Functions
EXPECTATION

Lesson: Limits of Polynomial, Rational, & Radical Functions

Learning Objective:

At the end of the learning episode, you are expected to:


1. apply the limit laws in evaluating the limit of algebraic functions:
polynomial, rational, and radical.

PRETEST

Give the correct answer.

1−5𝑥
1. Which of the following is the limit of 𝑓(𝑥) = 1−3𝑥 2+4𝑥 4 as 𝑥 approaches 1?
a. -2 B. 0 C. -6 D. Does not exist
2. Evaluate lim √𝑥 + 4 .
𝑥→0
a. 2 B. 3 C. 4 D. 5
3. What is the limit of 𝑔(𝑥) = 2𝑥 − 4𝑥 + 1 as x approaches to −1 ?
3 2

a. −6 B. −5 C. −4 D. 1
√2𝑥+5
4. Evaluate lim .
𝑥→2 1−3𝑥
3 1 1
a. − 5 B. 0 C. 2 D. 6
5. What is the limit of 𝑔(𝑥) = 3𝑥 − 2𝑥 − 1 as x approaches to 0 ?
4

a. −6 B. −5 C. −4 D. −1

RECAP

We presented limit theorems in our previous lesson.

In the following statements, 𝒄 is a constant, and 𝒇 and 𝒈 are functions which


may or may not have 𝒄 in their domains.
1. The limit of a constant is itself. If k is any constant, then,
lim 𝑘 = 𝑘 .
𝑥→𝑐
2. The limit of 𝑥 as 𝑥 approaches to 𝑐 is equal to c.
lim 𝑥 = 𝑐
𝑥→𝑐
3. The Constant Multiple Theorem
lim 𝑘 ∙ 𝑓(𝑥) = 𝑘 ∙ lim 𝑓(𝑥) = 𝑘 ∙ 𝐿
𝑥→𝑐 𝑥→𝑐
4. The addition Theorem

lim(𝑓(𝑥) ± 𝑔(𝑥)) = lim 𝑓(𝑥) ± lim 𝑔(𝑥) = 𝐿 ± 𝑀


𝑥→𝑐 𝑥→𝑐 𝑥→𝑐

5. The Multiplication Theorem


lim(𝑓(𝑥) ∙ 𝑔(𝑥)) = lim 𝑓(𝑥) ∙ lim 𝑔(𝑥) = 𝐿 ∙ 𝑀
𝑥→𝑐 𝑥→𝑐 𝑥→𝑐
6. The Division Theorem
𝑓(𝑥) lim 𝑓(𝑥) 𝐿
lim 𝑔(𝑥) = 𝑥→𝑐 = , 𝑝𝑟𝑜𝑣𝑖𝑑𝑒𝑑 𝑀 ≠ 0
𝑥→𝑐 lim 𝑔(𝑥) 𝑀
𝑥→𝑐
7. The Power Theorem
𝑦
lim(𝑓(𝑥))𝑦 = (lim 𝑓(𝑥)) = 𝐿𝑦
𝑥→𝑐 𝑥→𝑐
8. The Radical/Root Theorem
𝑚 𝑚
lim √𝑓(𝑥) = 𝑚√lim 𝑓(𝑥) = √𝐿
𝑥→𝑐 𝑥→𝑐

LESSON

In this lesson, we will illustrate how to use these limit theorems to


evaluate the limits of polynomial, rational and radical functions.

We start with evaluating the limits of polynomial functions.

Example1. Determine lim(5𝑥 + 1)


𝑥→1

Solution: Applying the given theorems,


lim(5𝑥 + 1) = lim 5𝑥 + lim 1 Addition
𝑥→1 𝑥→1 𝑥→1
= 5 lim 𝑥 + 1 Constant Multiple
𝑥→1

= 5(1) + 1 lim 𝑥 = 𝑐
𝑥→𝑐

=6
Therefore,
lim(5𝑥 + 1) = 6
𝑥→1
Example 2. Determine lim (4𝑥 3 − 3𝑥 2 + 1)
𝑥→−1
Solution: Applying the given theorems,
lim (4𝑥 3 − 3𝑥 2 + 1) = lim 4𝑥 3 − lim 3𝑥 2 + lim 1 Addition
𝑥→−1 𝑥→−1 𝑥→−1 𝑥→−1
= 4lim 𝑥 3 − 3lim 𝑥 2 + 1 Constant Multiple
𝑥→−1 𝑥→−1

= 4(−1)3 − 3(−1)2 + 1 Power

= −6
Therefore,
lim (4𝑥 3 − 3𝑥 2 + 1) = −6
𝑥→−1

We will now apply the limit theorems in evaluating rational functions.

𝑥
Example 3. Evaluate lim 𝑥−3
𝑥→2

𝑥 lim 𝑥 2
Solution: lim 𝑥−3 = 𝑥→2
= = −2 Division
𝑥→2 lim 𝑥−3 −1
𝑥→2
Therefore,
𝑥
lim = −2
𝑥→2 𝑥−3

3𝑥−1
Example 4. Evaluate lim
𝑥→−1 9𝑥 2 +5𝑥−2

3𝑥−1 lim (3𝑥−1)


Solution: lim = 𝑥→−1
𝑥→−1 9𝑥 2 +5𝑥−2 lim (9𝑥 2 +5𝑥−2)
𝑥→−1
lim (3𝑥)− lim 1
= 𝑥→−1 𝑥→−1
Addition
lim (9𝑥 2 )+ lim (5𝑥)− lim 2
𝑥→−1 𝑥→−1 𝑥→−1

𝟑 lim 𝑥− lim 1
= 𝑥→−1 𝑥→−1
Constant Multiple
9 lim 𝑥 2 + 5 lim 𝑥− lim 2
𝑥→−1 𝑥→−1 𝑥→−1

𝟑 lim 𝑥− lim 1
= 𝑥→−1
2
𝑥→−1
Power
9( lim 𝑥) + 5 lim 𝑥− lim 2
𝑥→−1 𝑥→−1 𝑥→−1

3(−1)−1 −4
= = = −2
9(−1)2 +5(−1)−2 2
Therefore,
3𝑥−1
lim = −2
𝑥→−1 9𝑥 2 +5𝑥−2
We will now evaluate limits of radical functions using limit theorems.

Example 5. Evaluate lim √𝑥 + 16


𝑥→0

Solution: lim √𝑥 + 16 = √lim(𝑥 + 16) Radical/Root Rule


𝑥→0 𝑥→0

= √ lim 𝑥 + lim16
𝑥→0 𝑥→0

= √0 + 16 = √16 = 4
Therefore,
lim √𝑥 + 16 = 4
𝑥→0

Example 6. Evaluate lim1 √9 + 4𝑥 2


𝑥→−
2

Solution: lim1 √9 + 4𝑥 2 = √ lim1 (9 + 4𝑥 2 ) Radical/Root Rule


𝑥→− 𝑥→−
2 2

= √ lim1 9 + lim14𝑥 2
𝑥→− 𝑥→−
2 2

= √ lim1 9 + 4 lim1 𝑥 2
𝑥→− 𝑥→−
2 2
2
= √ lim1 9 + 4 ( lim1 𝑥)
𝑥→− 𝑥→−
2 2

1
= √9 + 4(− 2)2

1
= √9 + 4(4) = √9 + 1

= √10
Therefore,
lim1 √9 + 4𝑥 2 = √10
𝑥→−
2

ACTIVITIES

Evaluate the following limits. Show your complete solutions and write it on
your notebook.

1 3
1. lim √𝑥 + 16 2. lim 3. lim √𝑥 2 + 3𝑥 − 6
𝑥→−7 𝑥→−2 𝑥 2 +5 𝑥→−2
3𝑥−2
4. lim 5. lim (8𝑥 4 + 2𝑥 3 − 𝑥 2 + 3𝑥 − 9)
𝑥→3 1−4𝑥 𝑥→−1

WRAP–UP

In evaluating the limits of polynomial, rational and radical functions,


you just simply apply the different limit theorems.

Can you state the different limit theorems?

VALUING

In mathematics, the limit of a function is a fundamental concept in calculus


and analysis concerning the behavior of that function near a particular input.
As a person, we should know our own limits. Knowing our limits will help us
know when to take a break.
Do you know your limits? How will you use it to your own advantage?

POSTTEST

Directions: Choose the letter of the correct answer by writing it on your


notebook.

4𝑥
1. Which of the following is the limit of 𝑓(𝑥) = 𝑥−5 as x approaches 2?
a. 2/5 B. 0 C. -8/3 D. Does not exist
3
2. Evaluate lim √𝑥 + 8 .
𝑥→0
a. 2 B. 3 C. 4 D. 5
3. What is the limit of 𝑔(𝑥) = 3𝑥 − 𝑥 + 5 as x approaches to −1 ?
3 2

a. −6 B. −5 C. −4 D. 1
√2𝑥−3
4. Evaluate lim .
𝑥→2 1−5𝑥
1 3 1 1
a. − 9 B. − 5 C. 2 D. 6
5. What is the limit of 𝑔(𝑥) = 7𝑥 − 8𝑥 + 5𝑥 − 10𝑥 − 1 as 𝑥 approaches to
4 3 2

0?
a. −6 B. −5 C. −4 D. −1
KEY TO CORRECTION

D 5. -7 5.
A 4. -7/11 4.
B 3. -2 3.
A 2. 1/9 2.
A 1. 3 1.

PRETEST ACTIVITIES

REFERENCES

BOOK

Cuaresma, Genaro A. et al. 2004. Analytic Geometry and Calculus 1: A


Worktext for Math 26. Los Baños, Laguna: Institute of Mathematical
Sciences and Physics, University of the Philippines.
Department of Education-Bureau of Learning Resources. 2016. Precalculus
Learner's Material.

Leithold, Louis. 1989. College Algebra and Trigonometry. Addison Wesley


Longman Inc., reprinted by Pearson Education Asia Pte. Ltd., 2002.
Basic Calculus SENIOR
HIGH
SCHOOL

Module

4
Quarter 3
Limits of Exponential,
Logarithmic, and Trigonometric
Functions
EXPECTATIONS

Lesson: Limits of Exponential, Logarithmic, and


Trigonometric Functions

Learning Objective:

At the end of the learning episode, you are expected to:


1. compute the limits of exponential, logarithmic, and trigonometric
functions using tables of values.

PRETEST

Give the correct answer.

1. Which of the following is the lim 3𝑥 ?


𝑥→1

A. 1 B. 3 C. 6 D. 9

2. Evaluate lim 𝑒 𝑥 .
𝑥→2

A. 7.39 B. 8.39 C. 9 D. 10

3. Which of the following is the lim log 𝑥?


𝑥→10

A. 4 B. 3 C. 2 D. 1

4. Evaluate lim 2𝑥 .
𝑥→3

A. 4 B. 6 C. 8 D. 16

5. Evaluate lim cos 𝑥.


𝑥→𝜋

A. 1 B. -1 C..984 D. .998
RECAP

In your previous lesson, you discussed the one - sided limits.


-If 𝑥 approaches c from the left, or through values less than c, then we write
lim− 𝑓(𝑥).
𝑥→𝑐

- If 𝑥 approaches c from the right, or through values greater than c, then we


write lim+ 𝑓(𝑥).
𝑥→𝑐

If the left-hand limit and the right-hand limit both exist and are both
equal to L, the lim 𝑓(𝑥) exists and is equal to L.
𝑥→𝑐

If the left-hand limit and the right-hand limit exist but are not equal,
or if one of these exists while the other does not, then the two sided limit
does not exist.

LESSON

Exponential Functions

An exponential function is defined as

𝑓(𝑥) = 𝑏 𝑥 or 𝑓(𝑥) = 𝑒 𝑥 where 𝑏 > 0 𝑎𝑛𝑑 𝑏 ≠ 1 and 𝑥 is a real number.

Evaluating Limits of Exponential Functions

First, we consider the natural exponential function 𝑓(𝑥) = 𝑒 𝑥 , where 𝑒 is


called the Euler number, and has value of 2.718281828.

Example 1. Evaluate the lim 𝑒 𝑥 .


𝑥→0

Solution: 𝑥 𝑓(𝑥)
−1 0.36787944117
Construct the table of values for
−0.5 0.60653065971
𝑓(𝑥) = 𝑒 𝑥 . We start by approaching −0.3 0.74081822066
−0.01 0.99004983374
the number 0 from the left or through −0.001 0.99900049983
−0.0001 0.99990000499
the values less than but close to 0. −0.00001 0.99999000005
Base on the result from the table, lim− 𝑒 𝑥 = 1 .
𝑥→0
Now we consider approaching 0 from its
𝑥 𝑓(𝑥)
right or through values greater than but 1 2.71828182846
0.5 1.6487212707
close to 0. From the table on the right, as 0.3 1.349858808
the values of 𝑥 get closer and closer to 0, 0.01 1.01005016708
0.001 1.00100050017
the values of 𝑓(𝑥) get closer and closer to 0.0001 1.000100005
0.00001 1.00001000005
1. So, lim+ 𝑒 𝑥 = 1.
𝑥→0

Therefore, the lim 𝑒 𝑥 = 1.


𝑥→0

Evaluating Limits of Logarithmic Functions

We now consider the common logarithmic function 𝑓(𝑥) = log10 𝑥. Recall that

𝑓(𝑥) = log10 𝑥 = log 𝑥.

Example 2. Evaluate lim log 𝑥. 𝑥 𝑓(𝑥)


𝑥→1
0.1 −1
Solution: We will construct the table of 0.3 −0.5228787453
0.5 −0.30102999566
values for lim− log 𝑥. We first approach 0.99 −0.0043648054
𝑥→1
0.999 −0.00043451177
the number 1 from the left or through 0.9999 −0.00004343161
0.99999 −0.00000434296
the values less than but close to 1.

Base on the result from the table, as the values of 𝑥 get closer and closer to
1, the values of 𝑓(𝑥) get closer and closer to 0, so the lim− log 𝑥 = 0.
𝑥→1

Now we consider approaching 1 from


𝑥 𝑓(𝑥)
its right or through values greater 2 0.30102999566
1.5 0.17609125905
than but close to 1. We will construct 1.3 0.1139433523
1.01 0.00432137378
the table of values for lim+ log 𝑥. Base on 1.001 0.00043407747
𝑥→1
1.0001 0.00004342727
the result from the table, the values of 1.00001 0.00000434292

𝑓(𝑥) get closer and closer to 0, so the lim+ log 𝑥 = 0.


𝑥→1

Therefore, the lim log 𝑥 = 0.


𝑥→1
Evaluating Limits of Trigonometric Functions

Example 3. Evaluate lim sin 𝑥.


𝑥→0

Solution: We will construct the table 𝑥 𝑓(𝑥)


−1 −0.8414709848
of values for lim− sin 𝑥. We first −0.5 −0.4794255386
𝑥→0
−0.3 −0.2955202067
approach 0 from the left or through −0.1 −0.09983341665
−0.001 −0.00099999983
the values less than but close to 0. −0.0001 −0.00009999999
−0.00001 −0.00000999999
Base on the result from the table, as

the values of 𝑥 get closer and closer to 0, the values of 𝑓(𝑥) get closer and
closer to 0. In symbols, lim− sin 𝑥 = 0.
𝑥→0

Now we consider approaching 0 from 𝑥 𝑓(𝑥)


its right or through values greater than 1 0.8414709848
0.5 0.4794255386
but close to 0. We will construct the table 0.3 0.2955202067
0.1 0.09983341665
of values on the right for lim+ sin 𝑥. Base 0.001 0.00099999983
𝑥→0
0.0001 0.00009999999
on the result from the table, the values 0.00001 0.00000999999

of 𝑓(𝑥) get closer and closer to 0, so the lim+ sin 𝑥 = 0.


𝑥→0

As the values of 𝑥 from the left and from the right get closer and closer to 0,
the values of 𝑓(𝑥) get closer and closer to 0.

Therefore, lim sin 𝑥 = 0.


𝑥→0
ACTIVITIES

Evaluate the following limits by completing the table of values. Write your
answer to the nearest whole number/hundredths.

1. lim 5𝑥 = _____
𝑥→2

𝑥 𝑓(𝑥) 𝑥 𝑓(𝑥)
3 125 1.1 5.873094715
2.5 1.3
2.3 1.5
2.01 25.40561478 1.99 24.60086108
2.001 1.999
2.0001
lim 5𝑥 = _____ 1.9999
lim 5𝑥 = _____
𝑥→2+
2.00001 𝑥→2−
1.99999
2. lim log 𝑥 = _____
𝑥→4

𝑥 𝑓(𝑥) 𝑥 𝑓(𝑥)
5 0.6989700043 3.1 0.4913616938
4.5 3.3
4.3 3.5
4.01 0.6031443726 3.99
4.001 3.999
4.0001 3.9999 0.6020491338
4.00001 3.99999

lim log 𝑥 = _____ lim log 𝑥 = _____


𝑥→4+ 𝑥→4 −
WRAP–UP

Finding the limit of a given function using table of values, construct a table
of values for lim− 𝑓(𝑥) and lim+ 𝑓(𝑥) .
𝑥→𝑐 𝑥→𝑐

We can say

lim 𝑓(𝑥) = 𝐿 ,
𝑥→𝑐

if and only if

lim 𝑓(𝑥) = 𝐿 and lim 𝑓(𝑥) = 𝐿 .


𝑥→𝑐 − 𝑥→𝑐 +

VALUING

Limits allow us to study a number from afar. That is, we can study the
points around it so we can better understand the given value we want to
know.
Having limits helps us organize investments of our time, energy and
other resources.
As a senior high school student, what are your limitations in life?

POSTTEST

Evaluate the following limits by constructing the table of values.


1. lim log 𝑥
𝑥→1/2
2. lim cos 𝑥
𝑥→𝜋
1 𝑥
3. lim ( )
𝑥→1 2
reprinted by Pearson Education Asia Pte. Ltd., 2002.
Leithold, Louis. 1989. College Algebra and Trigonometry. Addison Wesley Longman Inc.,
Material.
Department of Education-Bureau of Learning Resources. 2016. Precalculus Learner's
Sciences and Physics, University of the Philippines.
Worktext for Math 26. Los Baños, Laguna: Institute of Mathematical
Cuaresma, Genaro A. et al. 2004. Analytic Gometry and Calculus 1: A
BOOK
R E F E R E N CE S
2. lim log 𝑥 = .6
𝑥→4
𝑥 𝑓(𝑥)
𝑥 𝑓(𝑥)
3.1 0.4913616938
5 0.6989700043
3.3 0.5185139399
4.5 0.6532125138
3.5 0.5440680444
4.3 0.6334684556
3.99 0.6009728957
4.01 0.6031443726
3.999 0.6019514041
4.001 0.6021685514
3.9999 0.6020491338
4.0001 0.6020708486
3.99999 0.6020589056
4.00001 0.6020610771
lim log 𝑥 =.6
𝑥→4− lim log 𝑥 = .6
𝑥→4+
1. lim 5𝑥 = 25
𝑥→2
𝑥 𝑓(𝑥)
𝑥 𝑓(𝑥)
3 125
1.1 5.873094715
2.5 55.90169944
1.3 8.103282983
2.3 40.51641492
1.5 11.18033989 PRETEST
2.01 25.40561478
1.99 24.60086108
2.001 25.04026834 1. B
1.999 24.95979641
2.0001 25. 00402392 2. A
1.9999 24.99597673
2.00001 25. 00040236 3. D
1.99999 24.99959764
lim 5𝑥 = 25 4. C
lim 5𝑥 = 25 𝑥→2+
𝑥→2− 5. B
ACTIVITIES
KEY TO CORRECTION
Basic Calculus SENIOR
HIGH
SCHOOL

Module

5
Limits of Some Quarter 3

Transcendental Functions
EXPECTATION

Lesson: Limits of Some Transcendental Functions

Learning Objective:

At the end of the learning episode, you are expected to:


sin 𝑡 1−cos 𝑡 𝑒 𝑡 −1
1. illustrate limits involving the expressions , , 𝑎𝑛𝑑 using
𝑡 𝑡 𝑡
tables of values.

PRETEST

𝑡
Complete the table of values to evaluate lim sin 𝑡 = _______
𝑡→0

𝑡 𝑓(𝑡) 𝑡 𝑓(𝑡)
−1 1
−.5 0.5
−.3 0.3
−.1 0.01
−.001 0.001
−.0001 0.0001
𝑡 𝑡
lim− lim+
𝑡→0 sin 𝑡 𝑡→0 sin 𝑡

RECAP

Finding the limit of a given function using table of values, construct a table of
values for lim− 𝑓(𝑥) and lim+ 𝑓(𝑥) .
𝑥→𝑐 𝑥→𝑐

We can say lim 𝑓(𝑥) = 𝐿 ,


𝑥→𝑐

if and only if

lim 𝑓(𝑥) = 𝐿 and lim 𝑓(𝑥) = 𝐿 .


𝑥→𝑐 − 𝑥→𝑐 +
LESSON

In this lesson, we will determine some special limits of the following


sin 𝑡 1−𝑐𝑜𝑠 𝑡 𝑒 𝑡 −1
functions: 𝑓(𝑡) = , 𝑔(𝑡) = , 𝑎𝑛𝑑 ℎ(𝑡) = .
𝑡 𝑡 𝑡

sin 𝑡
Let us start by evaluating the function 𝑓(𝑡) = .
𝑡

sin 𝑡
Example 1. Evaluate lim .
𝑡→0 𝑡

Solution:
𝑡 𝑓(𝑡)
Let us construct the table of values −1 0.84147099848
sin 𝑡 −.5 0.9588510772
for lim− . We start by approaching −.3 0.9850673555
𝑡→0 𝑡
−.1 0.9983341665
the number 0 from the left or through −.001 0.9999998333
−.0001 0.99999999983
values less than but close to 0.
sin 𝑡
Therefore, lim− = 1.
𝑡→0 𝑡

Now we consider approaching 0 from its


𝑡 𝑓(𝑡)
right or through values greater than but
1 0.8414709848
close to 0. From the table on the right, as 0.5 0.9588510772
0.3 0.9850673555
the values of 𝑡 get closer and closer to 0, 0.01 0.9999833334
0.001 0.9999998333
the values of 𝑓(𝑡) get closer and closer to 0.0001 0.9999999983
sin 𝑡
1. So, lim+ = 1.
𝑡→0 𝑡

sin 𝑡
Therefore, the lim = 1.
𝑡→0 𝑡

1−cos 𝑡
Let us consider the next function 𝑔(𝑡) = lim .
𝑡→0 𝑡

1−cos 𝑡
Example 2. Evaluate lim
𝑡→0 𝑡
Solution:

Let us construct the table of values 𝑡 𝑔(𝑡)


−1 −0.4596976941
1−cos 𝑡
for lim− . We start by approaching −.5 −0.2448348762
𝑡→0 𝑡
−.3 −0.1488783696
the number 0 from the left or through −.01 −0.0049999583
−.001 −0.0004999999
values less than but close to 0. −.0001 −0.000005
1−cos 𝑡
Therefore, lim− = 0.
𝑡→0 𝑡

Now we consider approaching 0 from its


𝑡 𝑔(𝑡)
right or through values greater than but
1 0.4596976941
close to 0. From the table on the right, as 0.5 0.2448348762
0.3 0.1488783696
the values of 𝑡 get closer and closer to 0, 0.01 0.0049999583
0.001 0.0004999999
the values of 𝑔(𝑡) get closer and closer to 0.0001 0.000005
1−cos 𝑡
0. So, lim+ = 0.
𝑡→0 𝑡

1−cos 𝑡
Therefore, the lim =0
𝑡→0 𝑡

𝑒 𝑡 −1
Let us now consider the next special function ℎ(𝑡) = lim .
𝑡→0 𝑡

𝑒 𝑡 −1
Example 3. Evaluate lim .
𝑡→0 𝑡

Solution:

Let us construct the table of values


𝑒 𝑡 −1 𝑡 ℎ(𝑡)
for lim− . We start by approaching −1 0.6321205588
𝑡→0 𝑡
−.5 0.7869386806
the number 0 from the left or through −.3 0.8639392644
−.01 0.9950166251
values less than but close to 0.
−.001 0.9995001666
𝑒 𝑡 −1 −.0001 0.9999500016
Therefore, lim− = 1.
𝑡→0 𝑡
Now we consider approaching 0 from its
𝑡 ℎ(𝑡)
right or through values greater than but
1 1.718281828
close to 0. From the table on the right, as 0.5 1.297442541
0.3 1.051709181
the values of 𝑡 get closer and closer to 0, 0.01 1.005016708
0.001 1.000500167
the values of ℎ(𝑡) get closer and closer to 0.0001 1.000050002
𝑒 𝑡 −1
1. So, lim+ = 1.
𝑡→0 𝑡

𝑒 𝑡 −1
Therefore, the lim =1
𝑡→0 𝑡

ACTIVITIES

Use your notebook to answer the following.

𝑡
Complete the table of values to evaluate lim 𝑒 𝑡−1 = _________.
𝑡→0

𝑡 ℎ(𝑡) 𝑡 ℎ(𝑡)
−1 1
−.5 0.5
−.3 0.3
−.01 0.01
−.001 0.001
−.0001 0.0001
𝑡 𝑡
lim− lim+
𝑡→0 𝑒 𝑡 −1 𝑡→0 𝑒 𝑡 −1

WRAP–UP

To determine the limits of some special functions, you construct a table of


values and approach the given value of c from the left or through values less
than but close to c. Construct another table of values approaching c from its
right or through values greater than but close to c. You can determine the
limit of the given special function.
VALUING

Setting limits will help you to achieve your goals. You should definitely
dream big and examine any subconscious limits that may be holding you
back. However, an important step in achieving your goals and turning your
dreams into reality is to set carefully chosen limits. After all, limits help to
define, give shape, and add substance to your dreams.
As a senior high school student, what will be your limits to improve
your quality of life?

POSTTEST

Evaluate the following limits by constructing the table of values.


1−cos(3𝑡)
1. lim 3𝑡
𝑡→0

𝑡 𝑓(𝑡) 𝑡 𝑓(𝑡)

1 − cos(3𝑡) 1 − cos(3𝑡)
lim− lim+
𝑡→0 3𝑡 𝑡→0 3𝑡

sin (2𝑡)
2. lim
𝑡→0 2𝑡

𝑡 𝑔(𝑡) 𝑡 𝑔(𝑡)

sin (2𝑡) sin (2𝑡)


lim− lim+
𝑡→0 2𝑡 𝑡→0 2𝑡
𝑡 ℎ(𝑡) 𝑡 ℎ(𝑡)
1 0.5819767069 −1 1.581976707
0.5 0.7707470413 −.5 1.270747041
0.3 0.8574887741 −.3 1.157488774
0.01 0.9950083333 −.01 1.005008333
0.001 0.9995000833 −.001 1.000500083
0.0001 0.9999500009 −.0001 1.000050001
𝑡 𝑡
lim =1 lim =1
𝑡→0+ 𝑒 𝑡 −1 𝑡→0− 𝑒 𝑡 −1
𝑡
lim =1
𝑡→0 𝑒 𝑡 −1
ACTIVITY
𝑡
Complete the table of values to evaluate lim sin 𝑡 = __1_____
𝑡→0
𝑡 𝑓(𝑡) 𝑡 𝑓(𝑡)
1 1.188395106 −1 1.188395106
0.5 1.042914821 −.5 1.042914821
0.3 1.015159009 −.3 1.015159009
0.01 1.000016667 −.1 1.001668613
0.001 1.000000167 −.001 1.000000167
0.0001 1.000000002 −.0001 1.000000002
𝑡 𝑡
lim lim
𝑡→0+ sin 𝑡 𝑡→0− sin 𝑡
PRETEST
KEY TO CORRECTION
Basic Calculus SENIOR
HIGH
SCHOOL

Module

6
Quarter 3
Continuity of Functions at
a Point
EXPECTATIONS

Lesson: Continuity of Functions at a Point

Learning Objectives:

At the end of the learning episode, you are expected to:


1. illustrate continuity of a function at a number, and
2. determine whether a function is continuous at a number or not.

PRETEST

Give the correct answer.

Determine if the given functions 𝑓(𝑥) are continuous or not at the given

value of 𝑥.

1. 𝑓(𝑥) = 3𝑥 2 + 2𝑥 + 1 at 𝑥 = −2

Answer:__________________

2. 𝑓(𝑥) = 9𝑥 2 − 1 at 𝑥 = 1

Answer:__________________
1
3. 𝑓(𝑥) = 𝑎𝑡 𝑥 = 2
𝑥−2

Answer:__________________
𝑥−1
4. 𝑓(𝑥) = 𝑎𝑡 𝑥 = 1
𝑥 2 −1

Answer:__________________

√4−𝑥
5. 𝑓(𝑥) = 𝑎𝑡 𝑥 = 1
𝑥

Answer:__________________
RECAP

0
Let us recall the definition of indeterminate form of type .
0

0
A limit that is indeterminate of type may exist. To find the actual value,
0
one should find an expression equivalent to the original, by factoring or by
rationalizing. So, the expression that will emerge after factoring or
rationalizing will have a computable limit.
𝑥 2 −𝑥−2
Let us try to evaluate lim .
𝑥→−1 𝑥+1

Solution: The limit of both the numerator and the denominator as 𝑥


0
approaches -1 is 0. Thus, this limit is an indeterminate form of type .
0
However, observe that (𝑥 + 1) is a factor common to the numerator and the
denominator, and
𝑥 2 −𝑥−2 (𝑥−2)(𝑥+1)
= = 𝑥 − 2.
𝑥+1 (𝑥+1)

Therefore,
𝑥 2 −𝑥−2
lim = lim (𝑥 − 2) = −3.
𝑥→−1 𝑥+1 𝑥→−1

LESSON

In your previous discussion, there are functions whose limits are not equal
to the function value at 𝑥 = 𝑐, meaning, lim𝑓(𝑥) ≠ 𝑓(𝑐).
𝑥→𝑐

This leads us to the study of continuity functions. In this module, we will be


focusing on the continuity of a function at a specific point.

Once you trace the entire graph of the given function without lifting your
pen or pencil from your sheet of paper, the graph describes a continuous
function. Let us start by graphically illustrating what it means to be
continuity at a point.
Consider the graph below:

Let us use the graph to check if the


function is continuous at 𝑥 = 1. Note
that one is able to trace the graph from
the left side of the number 𝑥 = 1 going
to the right side of 𝑥 = 1, without lifting
one’s pen. Hence, we can say that the
function is continuous at 𝑥 = 1.

Let us have another example:


1
Consider the graph of the function 𝑓(𝑥) = .
𝑥−4

Is the function continuous at 𝑥 = 4?

If we trace the graph from the left of


𝑥 = 4 going to the right of 𝑥 = 4, we
have to lift our pen since at the left
of 𝑥 = 4, the function values will go
downward indefinitely, while at the
right of 𝑥 = 4, the function values
will go upward indefinitely.
Thus, the function is discontinuous
at 𝑥 = 4.

Suppose we are not given the graph of a function but just the function itself.
In this case, we have to check three conditions. Here are the three
conditions of continuity.

A function 𝑓(𝑥) is said to be continuous at 𝑥 = 𝑐 if the following three


conditions are satisfied:

(i) 𝑓(𝑐) exist;


(ii) lim 𝑓(𝑥) exist; and
𝑥→𝑐
(iii) 𝑓(𝑐) = lim 𝑓(𝑥)
𝑥→𝑐

If at least one of these conditions is not met, 𝑓 is said to be discontinuous


at 𝑥 = 𝑐.
Example 1. Determine if 𝑓(𝑥) = 𝑥 3 + 𝑥 2 − 2 is continuous or not at 𝑥 = 1.

Solution: We have to check the three conditions for continuity of a


function.

(a) If 𝑥 = 1, then 𝑓(1) = 13 + 12 − 2

= 1+1−2

=0

(b) lim 𝑓(𝑥) = lim 𝑥 3 + 𝑥 2 − 2


𝑥→1 𝑥→1
= 13 + 12 − 2

=0

(c) 𝑓(𝑐) = lim 𝑓(𝑥)


𝑥→𝑐

0 = 0 , Therefore, 𝑓 is continuous at 𝑥 = 1.
𝑥−3
Example 2. Determine if 𝑓(𝑥) = 𝑥 2 +𝑥−12 is continuous or not at 𝑥 = 3.

Solution:
3−3
(a) If 𝑥 = 3, then 𝑓(3) = 32+3−12
0
= 0
𝒇(𝒄) does not exist
Note that the given function is not defined at 𝑥 = 3, since 3 is not in the
domain of 𝑓. Hence, the first condition in the definition of a continuous
function is not satisfied. Therefore, 𝑓 is discontinuous at 𝑥 = 3.

Example 3. Determine if 𝑓(𝑥) = √𝑥 − 1 is continuous or not at 𝑥 = 4.


Solution: Let us check the three conditions.
(a) If 𝑥 = 4, then 𝑓(4) = √4 − 1

= √3 , since it is greater than 0,

𝒇(𝒄) exist.

(b) lim 𝑓(𝑥) = lim √𝑥 − 1


𝑥→𝑐 𝑥→4
= lim √4 − 1
𝑥→4
= 𝑙𝑖𝑚 √3
𝑥→4
= √3
(c) 𝑓(𝑐) = lim 𝑓(𝑥)
𝑥→𝑐

√3 = √3 , Therefore, the function 𝑓 is continuous at 𝑥 = 4.

ACTIVITIES

Use your graphing notebook to answer the following.

Activity A. Given the graph on the side, determine if the function 𝑓(𝑥) is
continuous at the following values of 𝑥.
1. 𝑥 = −1
2. 𝑥 = −3
3. 𝑥 = −2

Activity B. Determine if the following functions are continuous at the given


value of 𝑥. Show your complete solution using the three conditions of
continuity.

1. 𝑓(𝑥) = 2𝑥 2 + 𝑥 − 1 𝑎𝑡 𝑥 = 1
2. 𝑓(𝑥) = 4𝑥 2 − 1 𝑎𝑡 𝑥 = 1
3. 𝑓(𝑥) = √𝑥 − 3 𝑎𝑡 𝑥 = 1

WRAP–UP

How will you determine if the function is continuous at a given number?


Need to Remember
Three conditions of Continuity

A function 𝑓(𝑥) is said to be continuous at 𝑥 = 𝑐 if the following three


conditions are satisfied:

(i) 𝑓(𝑐) exist;


(ii) lim 𝑓(𝑥) exist; and
𝑥→𝑐
(iii) 𝑓(𝑐) = lim 𝑓(𝑥)
𝑥→𝑐

If at least one of these conditions is not met, 𝑓 is said to be discontinuous


at 𝑥 = 𝑐.

VALUING

In mathematics, a continuous function is a function that does not have any


abrupt changes in value, known as discontinuities. More precisely,
sufficiently small changes in the input of continuous functions result in
arbitrary small changes in its output. If not continuous , a function is said
to be discontinuous.
In life, problems are what make life worth living. They help us adapt to
become tougher as we adapt to different situations. Just continue to live and
focus positively whatever problem you are facing because it has always a
solution. Therefore, never allow your challenges to stop you from fulfilling
your true potentials in life.
POSTTEST

Give the correct answer.

Determine if the given functions 𝑓(𝑥) are continuous or not at the given

value of 𝑥. Show your complete solution.

1. 𝑓(𝑥) = 𝑥 2 + 2𝑥 + 1 at 𝑥 = 2

Answer:__________________

2. 𝑓(𝑥) = 𝑥 2 − 16 at 𝑥 = 1

Answer:__________________

1
3. 𝑓(𝑥) = 𝑎𝑡 𝑥 = 3
𝑥−5

Answer:__________________
𝑥−2
4. 𝑓(𝑥) = 𝑎𝑡 𝑥 = 2
𝑥 2 −4

Answer:__________________

√4−𝑥
5. 𝑓(𝑥) = 𝑎𝑡 𝑥 = 0
𝑥

Answer:__________________
KEY TO CORRECTION

5. DISCONTINUOUS CONTINUOUS 5.
4. DISCONTINUOUS DISCONTINUOUS 4.
3. CONTINUOUS DISCONTINUOUS 3.
2. CONTINUOUS CONTINUOUS 2.
1. CONTINUOUS CONTINUOUS 1.
POSTTEST PRETEST

3. DISCONTINUOUS 3. DISCONTINUOUS
2. CONTINUOUS 2. CONTINUOUS
B. 1. CONTINUOUS A. 1. CONTINUOUS
ACTIVITIES

REFERENCES

BOOK

Cuaresma, Genaro A. et al. 2004. Analytic Geometry and Calculus 1: A


Worktext for Math 26. Los Baños, Laguna: Institute of Mathematical
Sciences and Physics, University of the Philippines.
Department of Education-Bureau of Learning Resources. 2016. Precalculus
Learner's Material.

Leithold, Louis. 1989. College Algebra and Trigonometry. Addison Wesley


Longman Inc., reprinted by Pearson Education Asia Pte. Ltd., 2002.
Basic Calculus SENIOR
HIGH
SCHOOL

Module

7
Continuity of a Function Quarter 3

on an Interval
EXPECTATION

Lesson: Continuity of a Function on an Interval

Learning Objective:

At the end of the learning episode, you are expected to:


1. illustrate continuity of a function on an interval.

PRETEST

Give the correct answer.

1. Using the given graph on the side,


determine if the function 𝑓 is continuous
on the following intervals.

a). (−1,1)

b). (−∞, 0)

c). (0, +∞)

1
2. Let 𝑔(𝑥) = . Determine if g(x) is continuous
𝑥−4
on each of the following intervals.
a). [−2,3]

b). (0,4]

c). (4, +∞)


RECAP

Let us recall how to determine if the given function is continuous or


discontinuous at a given number. Using the three conditions of continuity, a
function 𝑓(𝑥) is said to be continuous at 𝑥 = 𝑐 if the following three
conditions are satisfied:

(i) 𝑓(𝑐) exist;


(ii) lim 𝑓(𝑥) exist; and
𝑥→𝑐
(iii) 𝑓(𝑐) = lim 𝑓(𝑥)
𝑥→𝑐

If at least one of these conditions is not met, 𝑓 is said to be discontinuous


at 𝑥 = 𝑐.

LESSON

A function can be continuous on an interval. A function is said to be


continuous on an interval when the function is defined at every point on
that interval and undergoes no interruptions, jumps, or breaks. If some
function 𝑓(𝑥) satisfies these criteria from 𝑥 = 𝑎 to 𝑥 = 𝑏, for example, we say
that 𝑓(𝑥) is continuous on the interval [𝑎, 𝑏].

Here are the given concepts which are important in determining whether a
function is continuous at the end points of closed intervals.

One- Sided Continuity

A function f is said to be continuous from the left at 𝑥 = 𝑐 if


𝑓(𝑐) = lim− 𝑓(𝑥).
𝑥→𝑐

A function f is said to be continuous from the right at 𝑥 = 𝑐 if

𝑓(𝑐) = lim+ 𝑓(𝑥).


𝑥→𝑐

Continuity of Polynomial, Absolute Value, Rational and Square Root


Functions
(a) Polynomial functions are continuous everywhere.
(b) The absolute value function 𝑓(𝑥) = |𝑥| is continuous everywhere.
(c) Rational functions are continuous on their respective domains.
(d) The square root function 𝑓(𝑥) = √𝑥 is continuous on [0, ∞).
Here are some of the graphs of functions to illustrate continuity on an
interval.
Example1. Consider the graph of the function 𝑓 given on the side and
determine if the function 𝑓 is continuous on the following intervals.
1. (−2, 2) 2. (−∞, 0) 3. (0, +∞)

Solution:
1. We can trace the graph from the right
side of 𝑥 = −2 to the left side of 𝑥 = 2
without lifting the pen we are using. We
can say that the function 𝑓 is continuous
on the interval (−2, 2).

2. If you trace the graph from negative infinity


to the left side of 0, we will not lift our pen and so, 𝑓 is continuous on
(−∞, 0).

3. For the interval (0, +∞), you trace the graph from the right side of 0 to
any large number, and find that you will not lift your pen. Thus the
function f is continuous on (0, +∞).
1
Example 2. Let 𝑓(𝑥) = 𝑥−4. Determine if 𝑓 is continuous on each of the
following intervals:
1. [−3,2]
2. (0, 4)
3. [0, 5]
4. (5, ∞)
5. (2,7)
6. [−5, 8)

Solution:
1. We can trace the graph from 𝑥 = −3 going to 𝑥 = 2 without lifting the
pen. Thus, the function 𝑓 is continuous on the closed interval [−3,2].
2. We can trace the graph from the right side of 𝑥 = 0 to the left side of
𝑥 = 4 without lifting the pen we are using. We can say that the function
𝑓 is continuous on the interval (0, 4).
3. By observation, the function 𝑓 is not continuous on the closed interval
[0, 5] because we will lift our pen by tracing the graph.
4. For the open interval (5, ∞), we can trace the graph from the right side
of 5 to any large number without lifting the pen. Thus, the function 𝑓
is continuous on (5, ∞).
5. The function 𝑓 is not continuous on the open interval (2,7) because we
will lift our pen by tracing the graph.
6. The function 𝑓 is not continuous on the given interval [−5, 8)because we
will lift our pen by tracing the graph.

ACTIVITIES
Determine if the given function is continuous on each of the given
intervals.

𝑨. 𝑓(𝑥) = 3𝑥 3 + 4𝑥 2 − 𝑥 + 6

1. [−4, 4] _____________
2. (0, 9) _____________

3
𝑩. 𝑓(𝑥) = 𝑥−2

1. [5, 10] _____________

2. (−4, 10) _____________

C. 𝑓(𝑥) = |−3𝑥|

1. [1, 1] _____________

2. (0, +∞) _____________

D. 𝑓(𝑥) = √3𝑥 − 1

1. (1, 10) _____________

2. [−1, +∞) _____________

WRAP–UP

How do you find if a function is continuous on an interval?

A function is said to be continuous on an interval when the function is


defined at every point on that interval and undergoes no interruptions,
jumps, or breaks. If some function 𝑓(𝑥) satisfies these criteria from 𝑥 = 𝑎 to
𝑥 = 𝑏, for example, we say that 𝑓(𝑥) is continuous on the interval [𝑎, 𝑏]
Need to Remember
One- Sided Continuity

A function f is said to be continuous from the left at 𝑥 = 𝑐 if


𝑓(𝑐) = lim− 𝑓(𝑥).
𝑥→𝑐

A function f is said to be continuous from the right at 𝑥 = 𝑐 if

𝑓(𝑐) = lim+ 𝑓(𝑥).


𝑥→𝑐

VALUING

We say a function 𝑓 is continuous on an interval when the function is


defined at every point on that interval and undergoes no interruptions,
jumps, or breaks.
Now, in this time of pandemic, continue pursuing your goal even if
there are some interruptions along the way for you to achieve your goal in
life. Always remember this: “Success is not final; failure is not fatal: It is the
courage to continue that counts.”

POSTTEST

Directions: Write the correct answer on your notebook. Show the graph and
determine if the given function is continuous on each of the
given intervals.

1. 𝑓(𝑥) = 4𝑥 2 − 𝑥 + 6; (−∞, 0) ____________

4
2. 𝑓(𝑥) = 𝑥−5 ; [−5, 5] ____________

3. 𝑓(𝑥) = √𝑥 − 1; [1, +∞) ____________

4. 𝑓(𝑥) = − |−5𝑥| [−1, +∞) ____________


KEY TO CORRECTION

c. continuous c. continuous
b. discontinuous b. continuous
2. a. continuous 1. a. continuous
PRETEST

2. discontinuous 2. continuous
D. 1. continuous C. 1. continuous

2. discontinuous 2. continuous
B. 1. continuous A. 1.continuous
ACTIVITIES

R E F E R E N CE S

BOOK

Cuaresma, Genaro A. et al. 2004. Analytic Geometry and Calculus 1: A


Worktext for Math 26. Los Baños, Laguna: Institute of Mathematical
Sciences and Physics, University of the Philippines.
Department of Education-Bureau of Learning Resources. 2016. Precalculus
Learner's Material.

Leithold, Louis. 1989. College Algebra and Trigonometry. Addison Wesley


Longman Inc., reprinted by Pearson Education Asia Pte. Ltd., 2002.
Precalculus
SENIOR
HIGH
SCHOOL

Module

8
Quarter 3
Problems Involving
Continuity
EXPECTATION

Lesson: Problems Involving Continuity

Learning Objective:

At the end of the learning episode, you are expected to:


1. illustrate the intermediate and extreme value theorems; and
2. solve problems involving continuity of a function.

PRETEST

A. Identify if the equation 𝑓 (𝑥 ) = 𝑥 5 − 2𝑥 4 + 𝑥 3 − 3𝑥 2 − 𝑥 + 5 has a solution on


each given closed interval. Show your proof using the intermediate value
theorem.
1. [-2, -1] 2. [-1, 0] 3. [0, 1] 4. [1, 2] 5. [2, 4]
B. Sketch the graph of 𝑓 (𝑥 ) = 4 − 𝑥 2 and then find the absolute extreme values
of the interval [-3, 1].

RECAP

In the previous modules, you have learned about an important quality


of the graphs of mathematical functions – continuity. Graphically, continuity
means that the curve is unbroken and uninterrupted. You can move along
the curve, corresponding to several points in the graph, without lifting your
pen. There are also three conditions for continuity, including a defined value
of f at c, a defined limit of f as x approaches c, and the equality of these two
values. Finally, some functions are not always continuous, as they may have
removable, jump, or essential discontinuities.
LESSON

A. Intermediate Value Theorem (IVT)


If a function f is continuous on the closed interval [a, b] and if 𝑓(𝑎) ≠ 𝑓(𝑏),
then for any number k between 𝑓(𝑎) and 𝑓(𝑏), there exists at least one number c
between a and b such that 𝑓(𝑐) = 𝑘.

Figure 1

In Figure 1, the function f is continuous on [a, b] where 𝑓(𝑎) ≠ 𝑓(𝑏). For a point
𝑘 between 𝑓(𝑎) and 𝑓(𝑏), we observe that there are 3 numbers 𝑐1 , 𝑐2 , 𝑐3 between a and
b where the values of the function f are all equal to k.

Example 1: Is there a solution to 𝑓(𝑥) = 𝑥 3 − 𝑥 − 1 between 𝑥 = 1 and 𝑥 = 2?

Solution:

The first condition of the IVT is that the function must be continuous. Since
the function is a polynomial function, the function is continuous at every real
number and thus is defined on the closed interval [1, 2].

Now, 𝑓(1) = 13 − 1 − 1 = −1 and 𝑓(2) = 23 − 2 − 1 = 5. A solution of a function


corresponds to the values of x where 𝑓(𝑥) = 0. Therefore, because −1 < 0 < 5, then,
according to IVT, the function has at least one solution between 𝑥 = 1 and 𝑥 = 2.
1
Example 2: Let 𝑓(𝑥) = . Determine if the IVT applies to f on the closed interval [0,
𝑥+2
4] for 𝑘 = 1.

Solution:

Since f is a rational function, f is continuous everywhere except at -2. Since


−2 is not in the interval of [0, 4], f is continuous at [0, 4].
1 1 1 1
Now, 𝑓(0) = = and 𝑓(4) = = . Since 1 is not between 𝑓(0) and 𝑓(4),
0+2 2 4+2 6
the IVT fails to hold for the given function on the closed interval [0, 4] and with 𝑘 = 1.
Example 3: Does the function 𝑓(𝑥) = 𝑥 4 + 𝑥 3 − 3𝑥 2 + 2𝑥 − 4 have a solution on the
closed interval [-2, 2]?

Solution:

The given is a polynomial function which is continuous everywhere and is


defined on the closed interval [-2, 2].

𝑓(−2) = (−2)4 + (−2)3 − 3(−2)2 + 2(−2) − 4 = −12

𝑓(2) = (2)4 + (2)3 − 3(2)2 + 2(2) − 4 = 12

Since 𝑓(−2) < 0 < 𝑓(2), that is −12 < 0 < 12, then yes, the function has a
solution on [-2, 2].

B. Extreme Value Theorem (EVT)

If f is continuous at every point of a closed interval [a, b], then f assumes both
an absolute maximum value M and an absolute minimum value m somewhere in [a,
b]. That is, there are numbers 𝑐1 and 𝑐2 within [a, b], such that 𝑓(𝑐1 ) = 𝑚 and 𝑓(𝑐2 ) =
𝑀, and 𝑚 ≤ 𝑓(𝑥) ≤ 𝑀 for every other x in the interval.

The highest value of a function f on a given closed interval is called its absolute
maximum value, and its lowest value is called its absolute minimum value.
Collectively, these values are known as extreme values of f on a closed interval.

Example 4: Identify the extreme values of the function on


the closed interval [-2, 1] shown in Figure 2.

Solution: On the given interval, the graph’s lowest value is


1 which occurs on two values of x (𝑥 = −2 and 𝑥 = 0), and
its highest value is 4 which occurs when 𝑥 = 1. Thus, the
absolute minimum value of the function is 1 and the
absolute maximum value is 4.
Figure 2

Example 5: Identify the absolute extreme values of the function


𝑓(𝑥) = −𝑥 2 defined on the interval [-3, 2].

Solution: First, sketch the graph of the function on the given


interval, similar to the one in Figure 3. Refer to the table of
values below.
x -3 -2 -1 0 1 2
f(x) -9 -4 -1 0 -1 -4

Based on the figure, the function f has an absolute


maximum value of 0 and an absolute minimum value of -9 on Figure 3
the interval [-3, 2].
Example 6: At a school canteen, bottled water is priced P20.00 for students. At that
price, the canteen sells 200 items daily. For every P2.00 increase in price, there are
10 fewer students willing to buy the bottled water.

a. With this scheme, what will be the revenue function of the school canteen for the
sale of bottled water?

b. If the manager of the canteen decides to implement the P2.00 price increase daily
for 10 days, what is the maximum revenue?

Solution:

a. The revenue function is simply the product of the selling price and the quantity of
items sold. Without the P2.00 increase, the revenue of the canteen is fixed at P20.00
X 200 = P4,000.00 daily. Let x represent the number of times that the canteen
increases the price of bottled water.

𝑅(𝑥) = 𝑠𝑒𝑙𝑙𝑖𝑛𝑔 𝑝𝑟𝑖𝑐𝑒 𝑋 𝑛𝑢𝑚𝑏𝑒𝑟 𝑜𝑓 𝑖𝑡𝑒𝑚𝑠 𝑠𝑜𝑙𝑑

𝑅(𝑥) = (20 + 2𝑥)(200 − 10𝑥)

𝑅(𝑥) = −20𝑥 2 + 200𝑥 + 4,000

b. You are asked to determine the absolute maximum value of the function on the
interval [0, 10]. To answer the question, you can set up a table of values for the
function and sketch the graph.

x 0 1 2 3 4 5 6 7 8 9 10
R(x) 4000 4180 4320 4420 4480 4500 4480 4420 4320 4180 4000

If you observe the table above, the maximum revenue


of the canteen will happen after a P2.00 increase is done 5
times; thus, it will reach a total price of P30.00 If the
canteen decides to increase the price 10 times, the revenue
will be the same as that when sold by the original price.

ACTIVITIES

A. Tell whether each given function has a solution on the indicated closed
interval. Prove using the IVT.

𝑥−3
1. 𝑓(𝑥 ) = 5𝑥 2 − 𝑥 − 1; [1, 2] 2. 𝑓(𝑥 ) = 𝑥+5; [-2, 4]
B. Sketch the graph of the following functions and then find the absolute
extreme values of each of the given interval.
2 𝑥−3
1. 𝑓(𝑥 ) = 3 − 5 𝑥; [-4, 0] 2. 𝑓(𝑥 ) = 4+𝑥; [-3, 1]

C. Let 𝑓 (𝑥 ) = 2𝑥 + 1. Determine if the IVT applies to f on the closed interval


[-3, 4] for 𝑘 = 1.

WRAP–UP

As a wrap up, answer the following:

1. What is the idea behind the Intermediate Value Theorem? How do we


state the theorem formally?

2. What is the Extreme Value Theorem? How do we use EVT to illustrate


a solution to real-life problem?

VALUING

In real-life, the intermediate-value theorem states that we must go to


places and cross borders to connect with the people we value the most. What
do you think is the most difficult border you have to cross to get into the
hearts of the ones you love most?
Recall an experience when your life was at its lowest point. How did you
feel? What did you do with what you feel?
The extreme value theorem tells you that within the closed interval on
which it is defined, a continuous function would have the highest and the
lowest values. Similarly, life is a mixture of highs and lows. You need to know
how to appreciate the highs and manage the lows.
POSTTEST

A. Tell whether each given function has a solution on the indicated closed
interval. Prove using the IVT.

2−𝑥 2
1. 𝑓(𝑥 ) = 3𝑥 2 + 2𝑥 2 ; [-1, 1] 2. 𝑓(𝑥 ) = ; [-3, -1]
𝑥2

B. Sketch the graph of the following functions and then find the absolute
extreme values of each of the given interval.
1
1. 𝑓(𝑥 ) = √𝑥 2 − 25; [5,10] 2. 𝑓(𝑥 ) = − 𝑥2 ; [0.5, 2]

C. A restaurant’s profit function (in hundreds) for hamburgers is given by


𝑥2
the function P such that 𝑃(𝑥 ) = 1.22𝑥 − 30,000 − 4,000, where 0 ≤ 𝑥 ≤ 20,000.
1. How many hamburgers does the restaurant need to sell to yield the
maximum profit?
2. What is the maximum profit from the sale of hamburger?
KEY TO CORRECTION
SENIOR
Basic Calculus HIGH
SCHOOL

Module

9
Quarter 3

Introduction
to Derivatives
EXPECTATIONS

LESSON: Introduction to Derivatives


Learning Objectives:

At the end of the learning episode, you are expected to


1. illustrate the tangent line to the graph of a function at a given
point.
2. relate the derivative of a function to the slope of the tangent line.

PRETEST

Direction: Choose the letter that corresponds to the correct answer.

Use the equation g ( x ) = x 2 + 2 in answering questions nos. 1-4

1. At what point does the graph intersect at x = 2 ?


a. 4 b. 6

2. What is the slope of the graph at x = 2 ?


a. 4 b. 2

3. What is the equation of the line at x = 2 ?


a. y = 4 x − 2 b. y = 4 x − 14
4. Which of the following refers to a line that intersect the graph at
exactly one point?
a. secant b. tangent

5. Which of the following is the formula in finding the slope of a tangent


line at a given point?
f ( x0 + x ) − f ( x0 ) f ( x0 + x) − f ( x0 )
a. ml = lim b. ml = lim
x→0 x x→0 x0
RECAP

In your junior high lesson, slope of a line (m) were introduced and
defined as the rise over run or the ratio between change of y and change of
x as shown in the given formula.
y1 − y2
m= , x1  x2
x1 − x2
Direction: Choose the correct letter that corresponds to the correct answer.
1. What is the sign of the slope when the direction of a line tends to the
right?
A. positive B. negative
2. What is the slope of the line when it is parallel to the y -axis?
A. zero slope B. undefined
3. What is the slope of a line that passes through point A (4,-8) and point
B(9,2)?
A. 2 B. -2
4. Which of the following is the slope of the line passing through (-5,6) and
(-3,2)?
A. 2 B. -2
5. Using the figure, what is the slope of a line
6 6
A. − B.
7 7
LESSON

In Geometry tangent line is defined as a line that touches the circle at


exactly one point as shown.

Line is tangent to circle A at point P. The


point at which the circle and the line intersect
(point P) is the point of tangency.

While tangent line to a curve starts with a secant line

By inspection, x is called increment it represents change in the value of x .


If the value of Q approaches P, x will also approach to 0 as shown in the
figure below.
Eventually line PQ will approach line l then if Q gets closer and closer to P ,
the slope of line l will be defined as :

The tangent line to the graph of f at the point P ( x0 , f ( x0 )) is the line


through P having the slope ml given by
f ( x0 + x ) − f ( x0 )
ml = lim ,
x→0 x

if this limit exists;


The line x = x0 , if

f ( x0 + x ) − f ( x0 )
lim is +  or − 
x→0+ x
and
f ( x0 + x ) − f ( x0 )
lim− is +  or − 
x→0 x

Illustrative Examples
1. Find an equation of the tangent line to the curve y = x 2 − 1 at point (2,3).

Solution: Solve the slope of the tangent line at (2,3)

Step 1: Solve for the slope of the line

f ( 2 + x ) − f ( 2 )
m( 2) = lim
x→0 x

= lim
 f (2 + x) − 1− 3 2

x
( 2 + x ) − 1− 3
x→0
2
= lim
x→0 x
Thus, the slope (m) is 4 at point (2,3).
4 + 4x + (x ) − 4
2
= lim
x→0 x Step 2: Use the point slope from to
determine the equation
4x + (x )
2
= lim y − y1 = m(x − x1 )
x→0 x
y − 3 = 4(x − 2)
x( 4 + x )
= lim
x→0 x y − 3 = 4x − 8

= lim(4 + x ) y = 4x − 5
x→0

m( 2 ) = 4
1
2. Give the equation of the tangent line to the graph of f ( x ) = at x = 1 .
x

Solution:

1
f (1) =
x

f (1) = 1

So the point intersect the graph of


1
f ( x ) = at (1,1).
x
Step 1: Solve for the slope of the line

f (1 + x ) − f (1)
m(1) = lim
x→0 x
f (1 + x ) − f (1)
m(1) = lim Thus, the slope (m) is -1 at point
x→0
1 x
−1 (1,1).
= lim 1 + x
x→0 x Step 2: Use the point slope from to
determine the equation
1 − (1 + x )
= lim 1 + x y − y1 = m(x − x1 )
x→0 x
y −1 = −1(x −1)
− x
= lim y −1 = −x +1
x→0 x(1 + x )

y = −x + 2
−1
= lim
x → 0 1 +  x

= −1

ACTIVITIES

A. Find the slope and equation of the tangent line to the graph of the
equation at a given point.
1. y = 2 x 2 + 4 x at point (-2,0)
2. y = 2 x3 + 3 at point (1,5)
3. y = 9 − x 2 at x = 2
WRAP-UP

What is the formula to use in finding the slope of the tangent at a


given equation and a point?

What are the steps in finding the equation of a tangent line to the
graph of a function at a given point?

VALUING

The direction of the line dictates the sign of slope whether it is


positive, negative, zero or undefined. We human give different emotions
based on turns of daily events or happenings in our life. It is so because that
is the gift of life and what is important is at the end of the day we go back to
our right senses and decide what is good and fair to all.
As a student, cite some situations where you react positively,
negatively, no reaction or you don’t know what to react.
POSTTEST

Direction: Choose the letter that corresponds to the correct answer.

1. Which of the following refers to a line that intersects the graph at


exactly one point?
a. secant b. tangent
2. Which of the following is the formula in finding the slope of a tangent
line at a given point ?
f ( x0 + x ) − f ( x0 ) f ( x0 + x) − f ( x0 )
a. ml = lim b. ml = lim
x→0 x x→0 x0
3. At what point the graph intersect g ( x ) = x 2 + 2 at x = 2 ?
a. 4 b. 6
4. What is the slope of g ( x ) = x 2 + 2 at x = 2 ?
a. 4 b. 2
5. What is the equation of the line of g ( x ) = x 2 + 2 at x = 2 ?
a. y = 4 x − 2 b. y = 4 x − 14
KEY TO CORRECTION

PRETEST RECAP POSTTEST ACTIVITIES

1. b 1. a 1. b A 1. m = −4 , y = −4 x − 8
2. a 2. b 2. a 2. m = 6 , y = 6 x − 2
3. a 3. a 3. b 3. m = −4 . y = −4 x + 13
4. b 4. b 4. a
5. a 5. a 5. a

B.
1. 2. 3. .

References

Leithold, L. College Algebra and Trigonometry. Addison Wesley Longman


Inc., 1989. reprinted by Pearson Education Asia Pte. Ltd., 2002.

Lial, M.L., J Hornsby, and D.I Schneider. College Algebra and Trigonometry
and Precalculus. Addison-Wesley Educational Publisher, Inc., 2001.
Basic Calculus SENIOR
HIGH
SCHOOL

Module

10
Quarter 3

Introduction
to Derivatives
EXPECTATIONS

LESSON: Introduction to Derivatives


Learning Objectives:

At the end of the learning episode, you are expected to apply the definition of
the derivative of a function at a given number.

PRETEST

Direction: Choose the letter that corresponds to the correct answer.


1. It refers to the process of finding the derivative of a function.
a. differentiation b. integration
0
2. A process used if the differentiation produces indeterminate form .
0
a. factoring or rationalization b. division or multiplication

3. Which of the following is an example of indeterminate form?


1 0
a. b.
1 0

4. Which of the following is 𝑓′(−2) of the function 𝑓(𝑥) = 𝑥 2 − 3 ?


a. 4 b. -4

5. Which of the following is 𝑓′(1) the function 𝑦 = 𝑥 5 ?


a. 5 b. 4

RECAP

Direction: Choose the letter that corresponds to the correct answer.


1. Which of the following refers to a line that intersects the graph at
exactly one point?
a. tangent b. secant
2. Which of the following is the formula in finding the slope of a tangent
line at a given point?
f ( x0 + x ) + f ( x0 ) f ( x0 + x ) − f ( x0 )
a. ml = lim b. ml = lim
x→0 x x→0 x
3. At what point the graph intersect g ( x ) = x3 − 1 at x = 2 ?
a. 6 b. 7
4. What is the slope of g ( x ) = x − 1 at x = 2 ?
3

a. 12 b. 15
5. What is the equation of the line of g ( x ) = x3 − 1 at x = 2 ?
a. y = 12x − 17 b. y = 12x − 21

LESSON

The derivative of a function at y = f (x ) at x = xo tells us that f ' ( x0 ) is


the slope of the tangent line to the graph of the function at point
P( x0 , P( x0 )) .
x f ( x0 + x ) − f ( x0 )
f ' ( x0 ) = lim = lim
x→0 y x →0 x
provided the limit exists.

The derivative of a function f ( x ) denoted f ' ( x ) at any x in the domain of


the given function is defined as

x f ( x + x ) − f ( x )
f ' ( x ) = lim = lim
x→0 y x→0 x
Provided the limit exists.

The following are the different notations of derivatives when y = f (x )

df dy
y' , f '( x) , , ,
dx dx

 f (x) , D  f (x)
d d d
f , y , X
dx dx dx
Note that the process of solving the derivative is called differentiation.
The techniques in differentiating functions is the same with evaluating limits
0
and in indeterminate form , factoring and rationalization process could be
0
utilized.

Illustrative Examples

1. Find the derivative of f ( x ) = x 2 2. What is the derivative of y = x

Solution: Solution:

f ( x + x ) − f ( x ) f ( x + x ) − f ( x )
f ' ( x ) = lim f ( x ) = lim
x→0 x x→0 x

( x + x )2 − x 2 ( x + x) − x
f ' ( x ) = lim f ' ( x) = lim rationalization
x→0 x x→0 x

x 2 + 2x + (x ) − x 2 ( x + x ) − x ( x + x ) + x
2
f ' ( x) = lim f ' ( x ) = lim •
x→0 x x→0 x ( x + x ) + x

2x + (x )
2
f ' ( x) = lim ( x + x ) − x
f ' ( x ) = lim
x→0 x x→0 x( ( x + x ) + x)
x( 2 + (x )
f ' ( x ) = lim ( x )
x→0 x f ' ( x ) = lim
x→0 x( ( x + x ) + x)
f ' ( x) = lim2 + x
x→0 1
f ' ( x ) = lim
f '( x) = 2
x→0 ( x + x ) + x

1
f ' ( x ) = lim
x→0 2 x
What is the derivative of
Note that the domain of the
derivative is permissible for all values
of x  0

An alternate equation of derivative by setting x = x0 + x therefore x = x − x0 ,


similarly f (x ) − f ( x0 ) will be produced. The derivative of f at x0 is defined
to be provided that the limit exists
x f ( x) − f ( x0 )
f ' ( x) = lim = lim
x→0 y x →0 x − x0
provided that the limit exists then, The derivative of f at x0 is the slope of
the tangent line at f ( x0, f ( x0 )) , if it exists.

Illustrative Examples

1. f ( x ) = 5 x − 1 , solve for f ' (1) 2x


3. f ( x ) = , solve for f ' ( 2)
x +1
Solution:
Solution:
Using the given definition x0 = 1
Using the given definition x0 = 2
(5 x − 1) − 4
f ' (1) = lim
x→1 x −1 2x
−1
f ' (1) = lim x + 1
5x − 5 x→2 x −1
f ' (1) = lim
x→1 x −1
2x
−1 x + 1
5( x − 1)
f ' (1) = lim f ' (1) = lim x + 1 .
x→1 x −1 x→2 x −1 x +1

f ' (1) = lim5 2 x − ( x + 1)


x→1 f ' (1) = lim
x→2 ( x − 1)( x + 1)

f ' (1) = 5
x −1
f ' (1) = lim
x→2 ( x + 1)( x − 1)
2. f ( x) = 5 x , solve for f ' ( 0)
1
f ' (1) = lim
Solution: x→2 ( x + 1)

f ( x) − f ( x0 ) 1
f ' ( x) = lim f ' (1) =
x→0 x − x0 3

Using the given definition x0 = 1

5
x −0
f ' (0) = lim
x→0 x−0

5
x
f ' (0) = lim
x→0 x

1 1
f ' (0) = lim 4 → + 
x→0
5 0 
x

f ' ( 0) = 

The derivative undefined at x = 0


therefore, the limit does not
exists (DNE)
ACTIVITIES

A. Find the derivative of the given function at a given number


1. y = 3x 2 − 2 at 𝑓 ′ (1)
2. y = x 2 − 1 at 𝑓 ′(2)
2x
3. f ( x ) = , solve for f ' ( −2)
x +1
4. y = 2 x 2 − 2 x , solve for f ' ( 0)

WRAP-UP

What are the different notations of derivatives ?

What can you say to the limit of a certain function when the derivative
is undefined?

VALUING

We have different and alternative ways of solving derivative of a


function like the different modality of how you learn today compared to the
old one. What is important is that you are doing the best way that you can to
learn and adopt to our current situation.
In a sheet of paper, write 3 or 4 sentences about the learning attitude
or habits you are doing to cope up with our present condition.
POSTTEST

Direction: Choose the letter that corresponds to the correct answer.

1. It refers to the process of finding the derivative of a function.


a. differentiation b. integration
2. What process can be used if the differentiation produces
0
indeterminate form 0 ?
a. factoring or rationalization b. division or multiplication
3. Which is 𝑓′(3) of 𝑓(𝑥) = 3𝑥 − 2x + 1?
2

a. 50 b. 52
1
4. Which is 𝑓′(−2) of 𝑓(𝑥) = ?
𝑥 −2
a. 1 b. 2
5. Which of the following is the sum of 𝑓′(1) and f (-1) of the function
f(x) = −2x 2 + 3x ?
a. 5 b. 6
KEY TO CORRECTION

PRE-TEST RECAP ACTIVITIES

1. a 1. a 1. 𝑓′(1) = 6
2. a 2. b 2. 𝑓′(2) = 4
3. b 3. b 3. 𝑓′(−2) = 2
4. b 4. a 4. 𝑓′(0) = −2
5. a 5. a

References

Leithold, L. College Algebra and Trigonometry. Addison Wesley Longman Inc.,


1989. reprinted by Pearson Education Asia Pte. Ltd., 2002.

Lial, M.J., J Hornsby, and D.I Schneider. College Algebra and Trigonometry
and Precalculus. Addison-Wesly Educational Publisher, Inc., 2001.
Basic Calculus SENIOR
HIGH
SCHOOL

Module

11
Quarter 3

Introduction
to Derivatives
EXPECTATIONS

LESSON: Introduction to Derivatives


Learning Objectives:

At the end of the learning episode, you are expected to determine the
relationship between differentiability and continuity of a function

PRETEST

Direction: Choose the letter that corresponds to the correct answer.


1. There are 4 conditions to determine if the given function 𝑓 is
continuous at the given number 𝑐 .
a. True b. False
2. If 𝑓 is continuous at an open interval 𝑐, then it is differentiable.
a. True b. False
3. Which of the given graph shows non differentiability?

a. b.

4. The polynomial function f(x) = 2x 2 − 1 differentiable at any number


in its domain.
a. True b. False
5. If the lim 𝑜𝑓 𝑓(𝑥) = 𝑓(𝑐) of a function 𝑓 then the differentiability fails.
𝑥→𝑐
a. True b. False
RECAP

A. Find the slope and equation of the tangent line to the graph of the
equation at a given point.
1. y = x3 − 2 x at point (-1,0)
2. y = x 2 − 3 at point (1,5)

LESSON

In our previous lesson we discussed that the process of solving


derivative is called differentiation. A process where 𝑓 ′ is solved from the given
function 𝑓 . A function is said to be differentiable function or continuous
function if it is differentiable in the open interval or every number in its
domain. It means that not all functions are differentiable in its domain.

The following are the 3 conditions of a given function 𝑓 is if continuous


at a given number (𝑐 )
1. 𝑓(𝑐) is defined
2. lim of 𝑓(𝑥) exists
𝑥→𝑐
3. lim 𝑜𝑓 𝑓(𝑥) = 𝑓(𝑐)
𝑥→𝑐

Note: The three conditions must hold, if not the given function is
discontinuous at the given value of 𝑐 .
If the given function 𝑓 is apolynomial function then the lim 𝑓(𝑐) = 𝑓(𝑐)
𝑥→𝑐
then it is differentiable.

Remarks:
1. If 𝑓 is discontinuous at 𝑥 = c,then f is not differentiable at 𝑥 = 𝑐.
2. If 𝑓 is continuous at 𝑥 = 𝑐, it does not mean that f is differentiable
at 𝑥 = 𝑐.
3. If 𝑓 is not differentiable at 𝑥 = c, it does not mean that f is not
continuous at 𝑥 = 𝑐.

A function 𝑓 can fail to be differentiable at a number 𝑐 if one of the reasons is


true:
1. The function 𝑓 is discontinuous at 𝑐.
2. The graph of the function 𝑓 has a vertical tangent at 𝑥 = 𝑐 .
3. The graph of the function 𝑓 has a sharp turn (corner or cusp).

Figure 1 Figure 2 Figure 3

Illustrative Examples:
Identify if the given function is differentiable at the given value of 𝑐
using the 3 conditions

1. Let 𝑓(𝑥) = 𝑥 1/3 , let 𝑥 = 0, using the stated conditions


1. 𝑓(0) is defined
𝑓(𝑥)−𝑓(𝑜)
2. 𝑓′(0) = lim
𝑥→0 𝑥−0
𝑥 1/3 −0
= lim
𝑥→0 𝑥
1
= lim 𝑥2/3
𝑥→0
The limit does not exist
3. lim 𝑓(0) ≠ 𝑓(0)
𝑥→0

Therefore, the given function is not differentiable but continuous at 𝑥 = 0. The


graph of this function does not have tangent line at the point where 𝑥 = 0.

2. Let 𝑓(𝑥) = |𝑥 |, let 𝑥 = 0, using the stated conditions


1. 𝑓(𝑐) is defined
𝑓(𝑥)−𝑓(𝑜)
2. 𝑓′(0) = lim 𝑥−0
𝑥→0
|𝑥|−0
= lim
𝑥→0 𝑥
|𝑥|
= lim
𝑥→0 𝑥

Since |𝒙|= x if 𝒙 > 𝟎 and |𝒙| = −𝒙 if 𝒙 < 𝟎,produces


two values of limits

|𝑥 | 𝑥 |𝑥 | −𝑥
lim = lim lim = lim
𝑥→0 𝑥 𝑥→0 𝑥 𝑥→0 𝑥 𝑥→0 𝑥

⬚ = lim 1 ⬚ = lim (− 1)
𝑥→0 𝑥→0
= 1 = −1
Because the two limits are not equal (two sided limits), therefore 𝑓 ′(0) does
not exist (DNE).
3. 𝑙𝑖𝑚𝑓(𝑥) ≠ 𝑓(0)
𝑥→0
Since the condition are not satisfied, therefore the given function 𝑓 is not
differentiable at 𝑥 = 0.

𝑥 2 , 𝑖𝑓 𝑥 < −1
3. Let 𝑓(𝑥) = { , solve if it differentiable at 𝑥 = −1, using the
−1 − 2𝑥, 𝑖𝑓 𝑥 ≥ 1
stated conditions
1. 𝑓(−1) is defined
Substitute 𝑓(−1) in 𝑓(𝑥) = −1 − 2𝑥 = 1

𝑓(𝑥)−𝑓(−1)
2. 𝑓′(−1) = lim
𝑥→0 𝑥−(−1)

Since the given is a piecewise function, we have to check the right hand
and left-hand limits

Left Hand Limit Right Hand limit


𝑓(𝑥) − 𝑓(−1) 𝑓(𝑥) − 𝑓(−1)
𝑓′(−1) = lim 𝑓′(−1) = lim
𝑥→−1 𝑥 − (−1) 𝑥→−1 𝑥 − (−1)
2
𝑥 −1 −1 − 2𝑥 − 1
= lim = lim
𝑥→−1 𝑥 − (−1) 𝑥→−1 𝑥 − (−1)

𝑥2 − 1 −2𝑥 − 2
= lim = lim
𝑥→−1 𝑥 + 1 𝑥→−1 𝑥 + 1

(𝑥 + 1)(𝑥 − 1) −2(𝑥 + 1)
= lim = lim
𝑥→−1 (𝑥 + 1)
𝑥→−1 (𝑥 + 1)
= lim 𝑥 − 1 = lim −2
𝑥→−1 𝑥→−1

= −2 = −2

3. 𝑙𝑖𝑚 𝑓(𝑥) ≠ 𝑓(−1)


𝑥→−1
−2 ≠ 1
Since the condition are not satisfied, therefore the given function 𝑓 is not
differentiable at 𝑥 = −1.
ACTIVITIES

𝑥 2 +2𝑥−3
, 𝑖𝑓 𝑥 ≠ 1
Using the 3 conditions, show that 𝑓(𝑥) = { 𝑥−1 is both continuous
4 , 𝑖𝑓 𝑥 = 1
and differentiable at 𝑥 = 1.

WRAP-UP

What are the 3 conditions to know if the given function 𝑓 is


continuous at a given value of 𝑐 ?

What are the three reasons a certain function 𝑓 fail to be differentiable


at the given value ?

VALUING

In our life, it is natural that we encounter problems that made us to


pause, stop and think to arrive at the right decision. These are problems that
will challenge our resiliency and strength. Always remember that there are
people like friends and family who are always willing to listen and help us
most especially when we feel we are alone.
As a student, how do you solve your problem triumphantly?
POSTTEST

Direction: Choose the letter that corresponds to the correct answer.

1. There are 3 conditions to determine if the given function 𝑓 is


continuous at the given number 𝑐 .
a. True b. False
2. If 𝑓 is continuous at an open interval 𝑐, then it is differentiable.
a. True b. False
3. The given graph shows differentiability

a. True b. False

4. The polynomial function f(x) = 2x 2 − 1 differentiable at any number


in its domain.
a True b. False
5. If the lim 𝑜𝑓 𝑓(𝑥) ≠ 𝑓(𝑐) of a function 𝑓 then the differentiability fails.
𝑥→𝑐
a. True b. False
KEY TO CORRECTION

PRETEST RECAP

1. b 1. m = 1 , y = x + 1
2. a 2. m = 2 , y = 2 x + 3
KEY TO CORRECTION
3. a
4. a
5. b

ACTIVITIES

References

Leithold, L. College Algebra and Trigonometry. Addison Wesley Longman Inc.,


1989. reprinted by Pearson Education Asia Pte. Ltd., 2002.

Lial, M.L., J Hornsby, and D.I Schneider. College Algebra and Trigonometry
and Precalculus. Addison-Wesley Educational Publisher, Inc., 2001.
SENIOR
Basic Calculus HIGH
SCHOOL

Module

12
Quarter 3

Derivative of Algebraic, Exponential


and Logarithmic Functions
EXPECTATIONS

Learning Objectives:

At the end of the lesson, you are expected to :

1) apply the differentiation rules in computing the derivative of algebraic


functions ; and

2) determine the derivative of exponential and logarithmic functions.

PRETEST

GIVEN : f(x)=2–x h ( x ) = 3x + 4
g ( x ) = 3x + 1 r ( x ) = 4x + 3
Find y’ if :
f(x) h (x)
1) y = 3) y =
g(x) r(x)
2) y = f ( x ) • g ( x ) 4) y = h ( x ) • r ( x )

RECAP

Consider the function defined by : f(x) = x2 if x  3


6x − 9 if x  3

Determine if each given statement is True or False. At x = 3, the function is

__________ 1) undefined
__________ 2) differentiable but not continuous
__________ 3) continuous but not differentiable
__________ 4) both continuous and differentiable
__________ 5) neither continuous nor differentiable
LESSON

In today's lesson, we will learn how to determine derivative in a more


manageable way and that is through the process of differentiation.
Differentiation is the process of finding the derivative. As the word imply,
differentiating means taking the derivative of a function. As such, we shall
call a function differentiable at c if f ' ( c ) exists.
The procedure of finding a derivative directly from a formula of the
function ( without referring to a graph ) is called Differential Calculus. In this
lesson, we will use some rules in finding the derivatives of Algebraic,
Exponential and Logarithmic Functions. We will start by getting familiar with
the symbols associated with the process of differentiation or finding the
derivative of a given function.

Symbols for Derivative

If y = f ( x ) ; then the first derivative of a function is symbolized by either of


the following :
o y’ - read as “y prime”
o f‘(x) - “f prime of x”
o dy - derivative of y with respect to x or dy over dx
dx
If u = f ( x ) then f ’ ( x ) is the same as du or
o du
dx
Some symbols may include other variables such as :
D ( x ), d , y , dx, dy , dA, ds, du, dv, dh, dz, etc.
dx x dt dt dt

DERIVATIVE OF ALGEBRAIC FUNCTIONS

Recall that determining the derivative of a simplest function using the


limit definition can be a lengthy and tedious process. What follows are rules
that will help us solve derivatives with less effort, we call them
Differentiation Rules/Formulas.

We first focus on the Differentiation Rules for Algebraic Functions :


Polynomial, Rational and Radical Functions
DIFFERENTIATION RULES

I. DERIVATIVE OF A CONSTANT FUNCTION :

The graph of a horizontal function is a horizontal line, and a horizontal


line has zero slope. Recall that the derivative measures the slope of the
tangent line, and so the derivative of a constant term is zero.

Example : y = 300,000 then the 1st derivative is y’ = 0

II. DERIVATIVE OF A POWER FUNCTION :


A function of the form : y = x n where n is a real number, is called
a power function. In general, this is called the POWER RULE :

If y = x n then, y' = nx n-1

Examples :
POLYNOMIAL FUNCTIONS
1) y = x
y' = (1) x1−1 = 1

2) If f ( x ) = x 3 ,
then f '( x ) = 3x 3-1 ; f '( x ) = 3x 2

RATIONAL FUNCTIONS
1
3) Find g'( x ), where g ( x ) =
x2
1
g(x) = can be rewritten as : g ( x ) = x -2
x2
g'( x ) = - 2x -2-1
2
g'( x ) = -2x -3 = −
x3

RADICAL FUNCTIONS
4
4) Find the derivative of y = √x 3

SOLUTIONS :

4
y = √x 3
3⁄
y = x 4 rewrite the expression to exponential form
3 3⁄ −1
y'= x 4 apply the power rule
4
3 −1⁄
= x 4 subtract the exponents
4
3 3
y' = 1 = simplify to radical form
4x ⁄4 4 4√x
1
5) Given : h ( x ) = 3 ; find h ' ( x )
√x

SOLUTIONS :

1
h(x) = 3
√x
1 −1⁄
h(x) = 1 =x 3 rewrite to exponential form
x ⁄3
1 −1⁄ −1
h'(x) = - x 3 apply power rule
3
1 4
= - x − ⁄3 subtract the exponents
3
1
= - 4 expressed in positive exponent
3x 3
1 1
h' ( x ) = - 3 = - 3 simplified radical expression
3 √x 4 3x √x

III. CONSTANT MULTIPLE RULE :

States that the derivative of a constant times a differentiable function


is the constant times the derivative of the function.

If y = k f ( x ) where k is constant ( k is the numerical coefficient of the


function of x ) ; then : y ‘ = k • f ‘ ( x )

EXAMPLES :
3 dy
1) Given : h ( x ) = 5x 4 ; find
dx

SOLUTION : Let y = h ( x )
3
y = 5x 4
3
y = 5 ⦁ x4 rewrite in the form k ⦁ xn
3 3
y ’ = 5 ⦁ 4 x 4 −1 apply constant multiple and power rule
15 1
= 4
x− 4 subtract exponents and combine similar terms
15
= 1 expressed in positive exponent
4𝑥4
15
y’ = simplest radical form
4 4√x

3
√x
2) Given : g(x)=
3
SOLUTION :

3
√x 1 1⁄
g(x)= ; g(x)= x 3 rewrite in the form k ⦁ xn
3 3
1
1 1
g’ ( x ) = ⦁ x 3−1 apply constant multiple and power rule
3 3
2

x 3
= subtract exponents and combine similar terms
9
1 1
g’ ( x ) = 2 = 3 in positive exponent to simplest radical form
9 x3 9 √x2

IV. SUM / DIFFERENCE RULE

Given two differentiable functions g and h, if y = g ( x )  h ( x ), then :


y ' = g' ( x )  h' ( x )

EXAMPLES :
3
1 3
Given : f ( x ) = 5x 4 ; g(x)=
3
√x and h ( x ) = − √3 x

From the given functions above find the following :


1) f ‘ ( x ) + g ‘ ( x )
2) g ‘ ( x ) - h ‘ ( x )
3) f ‘ ( x ) + h ‘ ( x ) – g ‘ ( x )

SOLUTIONS :
3 1 3
1) f ( x ) + g ( x ) = 5x 4 + √x combine the given functions by addition as indicated
3
3 1 1
= 5 • x4 + • x3 apply the constant multiple rule for each function
3
3 3 1 1 1
f ' ( x ) + g' ( x ) = 5 • x 4−1 + • x 3 −1 use the power rule
4 3 3
15 1 1 2
= • x −4 + • x − 3 subtract the exponents
4 9
15 1
= 1 + 2 expressed the terms with positive exponents
4x 4 9x 3
15 1
f ' ( x ) + g' ( x ) = 4 + 3 2 simplified radical expression
4 √x 9 √x

1 3
2) g ( x ) − h ( x ) = √x − (− √3 x ) combine the functions by addition as indicated
3
1 1
= • x 3 + √3 • x apply the constant multiple rule for each function
3
1 1 1
g' ( x ) − h' ( x ) = • x 3−1 + √3 ( 1 ) x 1-1 use the power rule
3 3
1 2
= • x − 3 + √3 • x 0 subtract the exponents
9
1
= 2 + √3 • 1 expressed the terms with positive exponents
9x 3
1
g ' ( x ) − h' ( x ) = 3 + √3 simplest radical form
9 √x 2

3 1 3
3) f ( x ) + h ( x ) − g ( x ) = 5x 4 + (−√3 x ) − √x
3
3 1
1
= 5 • x 4 − √3 • x − • x 3 apply the constant multiple rule
3
3 1
3 −1 1 1
f ' ( x ) + h' ( x ) − g' ( x ) = 5 • x 4 − √3 • x 1-1 − • x 3−1 use the power rule
4 3 3
1 2
15 −4 1 −3
= • x − √3 • x 0 − • x subtract the exponents
4 9
15 1
= 1 − √3 − 2 expressed with positive exponents
4 x4 9 x3
15 1
f ' ( x ) + h' ( x ) − g' ( x ) = 4 − 3 − √3 simplified radical form
4 √x 9 √x2

IV. PRODUCT RULE AND QUOTIENT RULE

Keep in mind that multiplying the derivative of one factor to the


derivative of another factor is NOT the rule in finding the product of two
given functions. Instead we are going to use this formula :

Given : y = f ( x ) • g ( x )
Let u = f ( x ) and v = g ( x ); then y = uv

Product Rule : y’ = udv + vdu

On the other hand, the derivative of a quotient is NOT equal to the


quotient of their derivatives. To determine the derivative of the quotient of
two functions we follow this rule :

f(x)
Given : y= ; where f ( x ) is the numerator and g ( x ) is the denominator
g(x)
u
Let u = f ( x ) and v = g ( x ) ; then y =
v

v du−u dv
Quotient Rule : y’ = v2

EXAMPLES :
4 1⁄
1) 𝑓 ( 𝑥 ) = ( √𝑥 3 ) ( 𝑥 3 + 2)
x-2
2) 𝑦 =
3x2
4 1⁄ 4 1⁄
1) f ( x ) = ( √x 3 ) ( x 3 + 2 ) ; Let u = √x 3 and v = x 3 + 2
SOLUTIONS :
4 1⁄ 1⁄ 4
f ' ( x ) = √x 3 • d ( x 3 + 2) + (x 3 + 2 ) • d ( √x 3 ) y ' = udv + vdu
3 1 1 1 3 3
f'(x) = x ⁄4 ( • x ⁄3−1 + 0) + ( x ⁄3 + 2)( • x ⁄4−1 ) power rule for each term
3 4
1 3⁄ 2⁄ 3 1⁄ 1⁄
= (x 4 • x− 3 ) + x− 4 (x 3 + 2) use distributive property
3 4
1 1 1
x ⁄12 3 x ⁄12 6x − ⁄4
= + + combine similar terms
3 4 4
13 3
f ' ( x ) = 12 12√x + simplest radical form
2 4√x

x-2 vdu - udv


2) y = 2 ; Let u = x - 2 and v = 3x 2 , Since y ' =
3x v2
SOLUTIONS :
3x 2 •d ( x - 2 ) - [ ( x - 2 ) •d ( 3x 2 )]
y' = (3x 2 )2
follow the formula
3x 2 ( 1 ) - [ ( x - 2 ) ( 3 •2x )]
y' = apply the power rule for each term
32 (x2 )2
2
3x - 6x ( x - 2 )
= simplify the expression
9x4
3x 2 - 6x 2
+ 12x
= use distributive property
9x4
- 3x 2 + 12x
= combine similar terms
9x4
-3x ( x - 4 ) −( x - 4 ) -x+4
y' = = or the derivative in simplest form
9x4 3x3 3x3

DERIVATIVE OF EXPONENTIAL AND LOGARITHMIC FUNCTIONS

In the previous study of the different function graphs you have learned
that exponential functions play an important role in modeling population
growth and the decay of radioactive materials. Logarithmic functions can help
rescale large quantities and are particularly helpful for rewriting complicated
expressions.

Just as when we found the derivatives of algebraic functions, we can also


find the derivatives of exponential and logarithmic functions using formulas.

DIFFERENTIATION FORMULAS : EXPONENTIAL AND LOGARITHMIC FUNCTIONS


Derivative of Exponential Functions :
If f ( x ) = e x , then f ' ( x ) = e x
If f ( x ) = b x , then f ' ( x ) = b x ln b
Derivative of Logarithmic Functions :
1
If f ( x ) = log 𝑏 x , then f ' ( x ) = x ln b
Derivative of Natural Logarithm :
1
If f ( x ) = ln x , then f ' ( x ) = x
EXAMPLES :
Determine the derivative of the following functions :

1) y = e-x

SOLUTIONS :

1
y = expressed with positive exponent
ex
dy ex • d ( 1) - [ 1 • d ( ex ) ]
= (ex )2
apply the quotient rule
dx
ex ( 0 ) - ( 1 ) ( ex )
= (ex )2
simplify the terms
dy -ex 1
= x x = - x derivative in simplest form
dx (e )(e ) e

2) f ( x ) = 2x • ex

SOLUTIONS : Let u = 2x and v = ex

f ( x ) = 2x • ex given
f ' ( x ) = 2x • d ( ex ) + ex • d ( 2x ) apply the product formula
= 2x ex + ex • 2x ln 2 combine similar terms
f ' ( x ) = 2x ex ( 1 + ln 2 ) factor and simplify

3) g ( x ) = x 2 + ( − 2 ln x )

SOLUTIONS :

g ( x ) = x 2 − 2 ln x clear off parenthesis


1
g ' ( x ) = 2x − 2 • apply the differentiation formula
x
2
= 2x − combine similar terms
x
2x 2−2
g'(x) = simplified into a single expression
x

4) h ( x ) = log 3 x − ( x − 1)2

SOLUTIONS : Since y = h ( x ), we follow the form h ( x ) = log b x

y = log 3 x − ( x 2 − 2x + 1 ) expand the given binomial


dy 1
= – ( 2x – 2 ) apply differentiation rules and formulas
dx x ln 3
dy 1
= – 2x + 2 simplify expression using distributive property
dx x ln 3
ACTIVITIES

ACTIVITY 1: Practice !
Apply differentiation rules in finding the derivative of the given
functions. Express your final answers in simplest form.
1
1) y = 3) g ( x ) = ( 2x )3
x5
2 1+ 4y3
2) f ( x ) = 4) x =
√x 3

ACTIVITY 2 : Keep Practicing !

DERIVATIVE OF A FUNCTION AT A NUMBER

Let f be a function and x = c is a value in its domain. The derivative of


f with respect to x evaluated at x = c is denoted by f ‘ ( c ). We first find the
derivative of the function using the differentiation rules, then we substitute
the value of c. Example : If g ( x ) = ln ( x ) ; solve for g′ ( 2 )
1 1
Solution : g’(x)= ; g’ ( 2 ) =
x 2

Now it’s your turn !


Find the indicated derivative :
3
1) f ( x ) = x 5 − 2x 2 + ; find f ’ ( -1 )
x
1
2) y = 2x 2 ; solve for y ’ ( 4 )
3t3 − 2t2 dx
3) x = ; determine if t = 0
2t2 dt

WRAP-UP

• What is the derivative of a constant function?

• Explain briefly, the basic power rule for differentiating algebraic


functions.

• Give the formula/s for differentiating exponential and logarithmic


functions.
VALUING

The 'd' in the derivative symbols stands for 'change'.

As an individual, each one of you would strive for ‘change’ to be the best
person that you can be. Bear in mind that just like 'derivatives' there are
rules to pursue to bring about these changes that you want to happen.

At present, what are the ‘rules to follow’ that you enjoy doing in the
hope of achieving these changes in your life?

POSTTEST

Multiple Choice: Write the letter of the correct answer on the space provided.
______ 1) The process of determining the derivative of a function is called __
a. differentiation c. derivation
b. difference d. power rule

3x3 − 2x2
______ 2) Which of the following is the derivative of y =
2x2
3 3
a. c. x
2 2
3
b. - 2 d. 0

______ 3) Which of the following rule will differentiate a polynomial function


1
a. If y = xn ; then f ( x ) = xn c. can be rewritten as x-n
xn
dy du
b. If y = xn ; then = nxn-1 d. Let u = f ( x ) ; so that f ’ ( x ) =
dx dx

1+4t3 dx
______ 4) If x = ; what is
3 dt
4
a. t c. 4t2
3
4
b. d. 4
3

______ 5) Determine the derivative of h ( x ) = 2 ln x


2
a. x −1 c. −
x
1 2
b. d.
x x
KEY TO CORRECTION

REFERENCES
BOOKS:
Department of Education - Bureau of Learning Resources, Basic Calculus
Teacher's Guide, ( pp. 127 - 137 ),First Edition 2016

Pelias JG., Basic Calculus, ( pp. 107 – 117 ), Rex Book Store Inc., First Edition
2016

WEB :
https://openstax.org/books/calculus-volume-1/pages/3-9-derivatives-of-
exponential-and-logarithmic-functions?©1999 -2020, Rice University.
Creative Commons Attribution 4.0

https://math.libretexts.org/Bookshelves/Calculus/Book%3A_Active_Calcul
us_(Boelkins_et_al)/1%3A_Understanding_the_Derivative/1.3%3A_The_Deri
vative_of_a_Function_at_a_Point

https://www.varsitytutors.com/high_school_math-help/finding-derivative-
at-a-point
SENIOR
Basic Calculus HIGH
SCHOOL

Module

13
Quarter 3

Derivative of Trigonometric Functions and


Inverse Trigonometric Functions
EXPECTATIONS

Learning Objectives :

At the end of the lesson, you are expected to :

1) apply the differentiation rules in computing the derivative of trigonometric


functions ; and

2) determine the derivative of inverse trigonometric functions.

PRETEST

MULTIPLE CHOICE : Write the letter of the correct answer.

1) If f ( u ) = – sec u , find f ‘ ( u )
a. – sec u tan u c. – sec x tan x
b. sec u tan u d. sec x tan x

dy
2) Find if y = 3 sin x – 2 cos x
dx
a. 3 cos x + 2 sin x c. 2 cos x + 3 sin x
b. –3 cos x + 2 sin x d. –2 cos x + 3 sin x


3) If f ( x ) = x sin x , then what is f ’ ( )
2
a. 1 c. –x
b. –1 d. x

4) Determine the derivative of y = x 2 tan x


x2 + x sin 2x x sin 2x− x2
a. − c.
cos2 x cos2 x
x2 − x sin 2x x2 + x sin 2x
b. d.
cos2 x cos2 x

dz 3w
5) What is , if z =
dw cos w
3 + w tan w 3 + 3w tan w
a. c.
cos w cos w
3 − 3w tan w 3 + 3w tan w
b. d. –
cos w cos w
RECAP

Determine if each given statement is True or False.

1) The derivative of 6x – 9 is 6x
2) If f ( x ) = x2 then f ' ( x ) = 2x
3) The function y = 2 ln x is not differentiable at x = 0
4) If f ' ( c ) is undefined then f ( x ) has no derivative
5) Differentiation is the process of finding the derivative of a function

LESSON

DERIVATIVE OF TRIGONOMETRIC FUNCTIONS

Let us recall the basic trigonometric functions and their reciprocals :

sin x ; csc x

cos x ; sec x
tan x ; cot x

The derivative formula that tells us how to get the derivative of a specific
algebraic function as given :
If f ( x )= x n then f ‘ ( x ) = nx n−1

We will derive a formula for the derivative of the function sin x. We begin
with using the principle in the limit definition of derivative which was
discussed in the previous lessons.

𝐟 ( 𝐱+ ∆𝐱 ) −𝐟 ( 𝐱 )
f ‘ ( x ) = lim
∆x→0 ∆𝐱
if the limit exists

sin ( x+ ∆x ) −sin x
For sin x we have : Dx ( sin x) = lim
∆x→0 ∆x
We use the sine sum identity sin ( A + B ) = sin A cos B + cos A sin B to get
sin x ( cos ∆x )+cos x sin ∆x − sin x
Dx ( sin x) = lim
∆x→0 ∆x
sin x ( cos ∆x −1 ) + cos x sin ∆x
Factor out sin x : Dx ( sin x) = lim
∆x→0 ∆x

sin x ( cos ∆x −1 ) cos x sin ∆x


Separate the two limit terms : = lim + lim
∆x→0 ∆x ∆x→0 ∆x

Since sin x and cos x are functions of x we can bring them outside the limit
cos ∆x −1 sin ∆x
expression to get : Dx ( sin x) = sin x [ lim ] + cos x [ lim ]
∆x→0 ∆x ∆x→0 ∆x

sin t cos t−1


• Recall the special limits : lim = 1 and lim =0
t →0 t t →0 t

cos t −1 sin t
If t = ∆x then : Dx ( sin x) = ( sin x ) lim
t
+ ( cos x ) lim
t→0 t →0 t

Substituting the value : Dx ( sin x) = sin x ( 0 ) + cos x ( 1 )


we get : 𝑫𝒙 ( 𝒔𝒊𝒏 𝒙) = cos x

The same principle and the same procedures were used to get the
derivative formula for cosine, however in the cosine formula we applied the
cosine sum identity : cos ( A + B ) = cos A cos B – sin A sin B
By algebraic manipulation, the equation will be reduced to
cos ∆x −1 sin ∆x
Dx (cos x) = cos x [ lim ] – sin x [ lim ]
∆x→0 ∆x ∆x→0 ∆x
cos t −1 sin 𝑡
If t = ∆x then : Dx (cos x) = ( cos x ) lim – ( sin x ) lim
t→0 t t →0 𝑡
Dx (cos x) = cos x ( 0 ) – sin x ( 1 )
Therefore : 𝑫𝒙 ( 𝒄𝒐𝒔 𝒙) = – sin x

• For the Tangent Function :


sin x
Apply the Quotient Identity : Dx ( tan x) = Dx [ ]
cos x
cos x ∙ Dx ( sin x ) − sin x ∙ Dx ( cos x )
Dx ( tan x) =
(cos x ) 2
cos x cos x − sin x ( −sin x )
Differentiation by Quotient Rule : = (cos x )2
cos2 x + sin2 x
Use the Pythagorean Identity : =
cos2 x
1
Dx ( tan x) =
cos 2 x

we simplify to : 𝑫𝒙 ( 𝒕𝒂𝒏 𝒙) = 𝒔𝒆𝒄𝟐 𝒙

DIFFERENTIATION FORMULAS FOR TRIGONOMETRIC FUNCTIONS:


If Dx stands for derivative, then :
Dx ( sin x ) = cos x
Dx ( cos x ) = – sin x
Dx ( tan x ) = sec2 x
Dx ( sec x ) = sec x tan x
Dx ( csc x ) = – csc x cot x
Dx ( cot x ) = – csc2 x
EXAMPLES :

1) f ( x ) = sec x + 3 csc x
SOLUTION : Applying the differentiation formulas
f ‘ ( x ) = sec x tan x + 3 ( – csc x cot x )
f ‘ ( x ) = sec x tan x – 3 csc x cot x

2) y = ex sin x
SOLUTION : Apply Product Rule and Formulas
dy
= ex ∙ Dx ( sin x ) + sin x ∙ Dx ( ex )
dx
dy
= ex cos x + ex sin x = ex ( cos x + sin x )
dx

3) y = x tan x
SOLUTION : Apply Product Rule and Differentiation Formulas
y’ = x ∙ Dx ( tan x ) + tan x ∙ Dx ( x )
= x sec2 x + tan x

DERIVATIVE OF INVERSE TRIGONOMETRIC FUNCTIONS

The derivative of an inverse of a function :

Suppose that on some interval [ a, b ] f has an inverse that is f −1 , where


f ( a ) ≠ f ( b ) for any y between f ( a ) and f ( b ). If x = f −1 ( y ), then
1
(f −1 ) y = . In the graphs of a function and its inverse, the points ( x, y )
f′ (x)
have a corresponding notation ( y, x ). If y = f −1 ( x ), the equivalent form is
dy 1
x = f ( y ). In other symbols : = dx
dx
dy

For inverse trigonometric functions, the angle whose sine is x is


represented by the symbol : sin−1 x, read as ′arcsine x′. That is, y = sin−1 x ; if
sin y = x and y = cos −1 x ; if cos y = x

DIFFERENTIATION FORMULAS FOR INVERSE TRIGONOMETRIC FUNCTIONS


1
Dx ( sin−1 x ) =
√1 − x2
1
Dx ( cos −1 x ) = –
√1 − x2
1
Dx ( tan −1
x)= 1+ x2
1
Dx ( cot −1 x ) = – 1+ x2
1
Dx ( sec −1 x ) =
x √x2 − 1
1
Dx ( csc −1
x)=–
x √x2 − 1
EXAMPLES :

1) y = x sin−1 x
SOLUTION :
1
y΄= x ⦁ + sin−1 x
√1 − x2
x
y΄= + sin−1 x
√1 − x2

2) f ( x ) = sec −1 x + csc −1 x
SOLUTION :
1 1
f΄( x ) = + [– ]
x √x2 − 1 x √x2− 1
1 1
f΄( x ) = – = 0
x √x2− 1 x √x2 − 1

ACTIVITIES

ACTIVITY 1 : Practice !

Find the derivative as indicated :


d
1) y = 3 cos x + 2 sin x ; find y΄ 3) f ( x ) = x 2 tan x ; find dx

; find f΄(
3π − 3t dx
2) f ( t ) = t sin t ) 4) x = ; find
2 cos t dt

ACTIVITY 2 : Keep Practicing !

Differentiate the following inverse trigonometric functions :


1) y = –2 sin−1 x 3) g ( u ) = – csc −1 u + sec −1 u
tan−1 w
2) s ( t ) = – cos −1 t 4) f ( w ) = – 3

WRAP-UP

• What are the differentiation formulas for finding the derivative of


trigonometric functions ?

• State the differentiation rues for the inverse trigonometric functions.


VALUING

There is a disciplinary value in the study of derivatives because it denotes


‘change’. Change in the quality of human life will be greatly influenced by how
each one of you will function as an individual being. Your ‘action’ or the
inverse, which is ‘inaction’ will determine how things will turn out.

Are you up for the challenge of whether to act or not to act on certain
situations ?

POSTTEST

MATCHING TYPE :

Write the letter of the correct derivative in Column B for the functions
given in column A.

COLUMN A COLUMN B
Functions Derivative

x2 +x sin 2x
_________ 1) – cos −1 x a. cos2 x

1−sin x
_________ 2) csc x + 3 sec x b. cos2 x

_________ 3) x 2 tan x c. 3 sec x tan x – csc x cot x

1 1
_________ 4) – cot −1 x d.
2 √1− x2

sin x −1 1
_________ 5) e.
cos x 2+2x2
KEY TO CORRECTION

REFERENCES
BOOKS :

Department of Education - Bureau of Learning Resources, Basic Calculus


Teacher's Guide, ( pp. 136 - 139 ), First Edition 2016

Pelias JG., Basic Calculus, ( pp. 133 – 136 ), Rex Book Store Inc., First Edition
2016

WEB :

https://openstax.org/books/calculus-volume-1/pages/3-9-derivatives-of-
exponential-and-logarithmic-functions?©1999 -2020, Rice University.
Creative Commons Attribution 4.0

https://christintheclassroom.org/vol_11/11cc_133-152.htm

https://www.mathsisfun.com/calculus/derivatives-trig-
proof.html?fbclid=IwAR0xf4_tv9ia-S8aA7V96V2hO2RXyZuBAxQS0Mj
SENIOR
Basic Calculus HIGH
SCHOOL

Module

14
Quarter 3

EXTREME VALUE THEOREM


EXPECTATION

Learning Objective :

At the end of the lesson, you are expected to illustrate the extreme value
theorem.

PRETEST

Direction : Determine whether the statement is true or false.

___________ 1. If a function is continuous on a closed interval [ a, b ], then it


has a maximum and a minimum on that interval.

___________ 2. If a function is discontinuous on a closed interval, then it has


no extreme value on that interval.

___________ 3. If a function has a maximum and a minimum over a closed


interval, then it is continuous on that interval.

___________ 4. If a function has no extreme values on [ a, b ], then it is


discontinuous on that interval.

___________ 5. If a function has either a maximum only or a minimum only


over a closed interval, then it is discontinuous on that interval.

RECAP

To find the critical points of a function, we differentiate f and find all values
of x such that f ‘ ( x ) becomes zero or undefined. Let us take this example :
Find the critical point/s of the function f ( x ) = 3x 2 − 3x + 4.
SOLUTION :
First, we differentiate the function and then equate f ‘ ( x ) to zero :
f ‘ ( x ) = 6x – 3 ; 0 = 6x – 3 therefore x = ½
So x = ½ is a critical point.
NOW IT’S YOUR TURN !

Find the critical points of the following functions :

1⁄
__________ 1) f ( x ) = –3 x 3 +x 3) f ( x ) = –10 + x3 – x – x2

__________ 2) f ( x ) = 2x4 – 8x2 4) f ( x ) = 4 + 3x 2 − 3x

LESSON

THE EXTREME VALUE THEOREM

Let f be a function which is continuous on a closed and bounded interval


[ a, b ]. Then the extreme values ( maximum and minimum ) of f always exist,
and they occur either at the endpoints or at the critical points of f.

A critical point of a function of a single real variable f ( x ), is a value x0 in


the domain of f where it is not differentiable or its derivative is 0, that is f ′ (x0)
= 0. To obtain this, we take the derivative of f and equate the result to zero.

We say that an extremum of f occurs at x if either the maximum or the


minimum occurs at x.

If a point is the highest among all the points in the graph, then it is
immediately the highest point among its neighboring points.

It is important to stress the assumptions of the Extreme Value Theorem.


The function f must be continuous on an interval that is closed and bounded.
To illustrate this, consider the following :

➢ A parabola f ( x ) = x 2 defined on R. It indeed has a minimum point at


the origin, but it does not have a maximum ( since the values go to
infinity as x approaches ± ∞ ). The conclusion of the Extreme Value
Theorem fails because even if f is continuous, the interval on which it
is defined is not bounded.

➢ The function f ( x ) = │ x │ if x є [ –1, 0 ) ∪ ( 0, 1 ] and if x = 0. Even if f


is defined on the closed and bounded interval [ –1, 1], the function is
not continuous there. So, the conclusion fails. As we see from the graph,
f has a maximum occurring at x = –1, 0, 1 but does not possess a
minimum because the value 0 is not attained.
To find the maximum or minimum values or points of a continuous
function on a given interval we may consider the following steps :

Find the derivative of the given function and then let f ‘ ( x ) = 0.

The values of x when f ‘ ( x ) = 0 are the critical points of the function.

Get the functional values at the endpoints and at the critical points.

Compare the values. The highest one is the maximum value while the
lowest one is the minimum value.

EXAMPLES :
Find the extrema of the given functions on the interval [ –1, 1 ] :

1) f ( x ) = 15x + x3 – 20 – 9x2

SOLUTION :

The critical points of f are 1 and 5, but since we limited our domain to
[ –1, 1 ], we are only interested with x = 1 and x = – 1.

➢ f ( x ) = 15x + x3 – 20 – 9x2 ; f ’ ( x ) = 15 + 3x2 – 18x


0 = 3x2 – 18x + 15
therefore : x = 1 ; x = 5

Below is the table of functional values at this critical point, as well as


those at the endpoints.

➢ We substitute to get these functional values :

x –1 1
f ( x ) – 45 – 13

➢ EXTREMA :
Therefore, the maximum value f ( 1 ) = –13 occurs at x = 1 while
the minimum value f ( –1 ) = – 45 occurs at x = –1. The maximum point
occurs at ( 1, – 13 ) and the minimum point is at ( – 1, – 45 )

2) f ( x ) = x – 3 3√x

SOLUTION :

1
➢ f ( x ) = x – 3 3√x ; f ( x ) = x – 3x 3
1
1
f ‘ ( x ) = 1 – 3 ( 3 ) x 3−1
2
0 = 1 – x− 3
1
3 =1 therefore : x=1 ;x=0
√x2
➢ functional values :

x 0 1 –1
f ( x ) 0 –2 2

➢ EXTREMA :
The functional values are f ( 0 ) = 0, f ( 1 ) = –2 and f ( –1 ) = 2.
The maximum point is ( –1, 2 ) while the minimum point is ( 1, – 2 )

3) f ( x ) = – x + x2

SOLUTION :

➢ f ( x ) = –x + x2 ; f ‘ ( x ) = – 1 + 2x
0 = – 1 + 2x
therefore : x = ½
➢ functional values :

X ½ 1 –1
f ( x ) –¼ 0 2

➢ EXTREMA :
The maximum point is ( –1, 2 ) while the minimum point is ( ½ , –¼ )

4) f ( x ) = 2x2 ( x2 – 4 )

SOLUTION :

➢ f ( x ) = 2x2 ( x2 – 4 ) ; f ( x ) = 2x4 – 8x2


f ‘ ( x ) = 8x3 – 16x
0 = 8x ( x2 – 2 )
therefore : x = 0, ± √2
➢ functional values :

x ±√2 0 –1 1
f ( x ) –8 0 –6 –6

➢ EXTREMA :
The maximum point is ( 0, 0 ) while the minimum points are :
( √2 , – 8 ) and ( – √2 , – 8 )
ACTIVITIES

ACTIVITY 1 : Practice !

Directions :
• Find the extrema of the following fuctions on the given interval.
• Determine the value of x at which the extrema occur.
1) f ( x ) = –3x 2 + 4x 3 + 3 − 6x on [ –1, 1 ]
2π π
2) f ( x ) = – cos x on [– , ]
3 3
x
3) f ( x ) = on [ –1, 2 ]
x2 +2

ACTIVITY 2 : Keep Practicing !

Answer the following problems :


1) Find the number in the interval [ –2, 2 ] so that the difference of the
number from its square is maximized.
2) Determine whether the function f ( x ) = sin x will have extrema
π π
(maximum and minimum) on the interval [ – , ]
2 2

WRAP-UP

• How do you find the extrema of a function, on a given interval ?

• State the Extreme Value Theorem

VALUING
Obtaining the maximum or the minimum benefit from the outcome of
your decisions will be affected by the choices you make as an individual. If
face with such critical situations seek the divine guidance of our God almighty
to help you come up with a sound judgement.

In your math journal, write a simple story that describes how you were
able to get to a critical decision in a certain situation.
POSTTEST

MULTIPLE CHOICE : Write the letter of the correct answer.


1) Which of the following is a critical point of the function f ( x ) = cos x ?
π
a. c.
2
π 3π
b. – d. –
2 2
2) Which of the following is the maximum point of the function
f ( x ) = x 2 − x on the interval [ –2, 2 ].
a. ( 2, 2 ) c. ( 2, 6 )
b. ( –2, 6 ) d. ( ½, –¼ )
3) Given the interval [ –2, 2 ] for the function f ( x ) = –8x2 + 2x4 ; at which
value/s of x will the minimum point occur ?
a. ± √2 c. –2
b. ±2 d. 2
4) The table shows the functional values for f ( x ) = sin x. What are the extrema
of the given function ?
π
a. ( – , –1 ) ; ( 0, 0 ) π π
2 x – 0
π 2 2
b. ( ,1);( ,0) sin x –1 0 1 0
2
c. ( , 0 ) ; ( 0, 0 )
π π
d. ( – , –1 ) ; ( , 1 )
2 2
5) Which of the given statements is true about the continuous function :
f ( x ) = x 2 , defined on R.
a. It has a minimum point at the origin
b. It has a maximum value
c. The function is not differentiable
d. ( 0, 0 ) is the maximum point
KEY TO CORRECTION

REFERENCES
BOOKS :

Department of Education - Bureau of Learning Resources, Basic Calculus


Teacher's Guide, ( pp. 144 - 149 ), First Edition 2016

Pelias JG., Basic Calculus, ( pp. 163 – 166 ), Rex Book Store Inc., First Edition
2016

WEB :

https://openstax.org/books/calculus-volume-1/pages/3-9-derivatives-of-
exponential-and-logarithmic-functions?©1999 -2020, Rice University.
Creative Commons Attribution 4.0

https://christintheclassroom.org/vol_11/11cc_133-152.htm

https://www.mathsisfun.com/calculus/derivatives-trig-
proof.html?fbclid=IwAR0xf4_tv9ia-S8aA7V96V2hO2RXyZuBAxQS0Mj

https://www.slideshare.net/gregcross22/ap-calculus-extrema-v2
Basic Calculus SENIOR
HIGH
SCHOOL

Module

15
SOLVING OPTIMIZATION
Quarter 3

PROBLEMS
EXPECTATION

Lesson: Solving Optimization Problems

Learning Objective:

At the end of the learning episode, you are expected to:


1. solve optimization problems that yield polynomial functions.

PRETEST

Directions. Read the problem carefully and then solve.

PROBLEM: A rectangular garden is to be constructed against the side of a


garage. The gardener has 100 feet of fencing, and will construct a three-
sided fence; the side of the garage will form the fourth side.

1. If the width of the garden is x feet, then the length of the garden is
____.
2. What is the area of the garden in terms of x?
3. What dimensions will give the garden of greatest area?

RECAP

Directions: Find the absolute maximum and minimum values of each of the
following functions continuous on a given interval.

1. f(x) = x2 on [-2, 1]
2. g(t) = 8t – t4 on [-2, 1]
3. f(x) = x2/3 on [-2, 3]

LESSON

Now that you have learned how to master finding derivatives, I’m sure
you can’t wait to put your expertise to use solving some practical problems.
In this lesson, you will find problems that actually come up in the real
world like the optimization problems.
One major application of calculus and considered as one of the most
practical types of problems is in optimization, where we are considering a
quantity. To “optimize" something means that we want to maximize or
minimize the quantity. Suppose there is a certain quantity we are interested
about which could be an amount of money, length of a fence, an angle,
shipping cost, or one of loads of other possibilities. If that quantity is a good
thing, like an amount of money you earned from work, then we would like
to make the quantity as large as possible; if it is not a good thing, like the
electric bills, then we'd like to make it as small as possible.
To guide you in solving optimization problems, follow the steps
written below.
STEPS IN SOLVING OPTIMIZATION PROBLEMS
1. Read and understand the problem.
2. Draw a diagram, if applicable.
3. Write down formulas and given information.
4. Write a function for what is to be optimized.
5. Set the derivative of the function equal to zero and solve.

Let us try to apply the abovementioned steps in solving the following


problems:

Example 1. Ramon wants to make an open – top box which is to be made


by cutting small congruent squares from the corners of a 18 inches by 18
inches metal sheet and bending up the sides. How large should the squares
cut from the corners be to make the box hold as much as possible?

Solution: We begin with the following figure.

In the figure, the corner squares are x inches. The volume of a rectangular
prism is 𝑉 = 𝑙 𝑒𝑛𝑔𝑡ℎ 𝑥 𝑤𝑖𝑑𝑡ℎ 𝑥 ℎ𝑒𝑖𝑔ℎ𝑡 and therefore the volume of the box can
be represented by the function
𝑉 (𝑥 ) = 𝑥 (18 – 2𝑥 )2
= 324𝑥 − 72𝑥 2 + 4𝑥 3
Since each of the sides of the sheet measures 18 inches, the value of 𝑥 ≤ 9
and therefore the domain of V is the interval 0 ≤ 𝑥 ≤ 9. The derivative of V in
terms of x:
𝑑𝑉
= 324 − 144𝑥 + 12𝑥 2
𝑑𝑥
= 12( 27 − 12𝑥 + 𝑥 2 )
= 12( 9 − 𝑥)(3 − 𝑥)
Of the two zeros, x = 3 and x = 9, only x = 3 lies in the interior of the
domain of the function and makes the critical -point list. The values of V(x)
at the given critical point and the two endpoints are as follows:

x 0 3 9
V( x ) 0 432 0

Thus, the maximum volume is 432 in3. The cutout squares should be 3
inches on a side.
Example 2. A farmer wants to build a rectangular pen adjacent to a horse
barn. The pen must be 2,700 square feet in area. Because of the need to
prepare the land, the side parallel to the barn will cost one and a half times
as much per foot to build as the two sides perpendicular to the barn. What
dimensions will produce a 2700-square-foot pen for the lowest cost?
Solution. Your plan is to minimize the cost of the fence. The area of the
rectangular pen can be represented as 𝑥𝑦 = 2700. Since the cost of side x is
one half times the cost of side y, then the cost function is
𝐶 = 1.5𝑥 + 2𝑦.
Rewrite the cost function in one variable by substituting from 𝑥𝑦 =
2700
2700 𝑜𝑟 𝑦 = . The new equation will be
𝑥

5400
𝐶 = 1.5𝑥 +
𝑥
The derivative of the given cost function is
5400
𝐶 ′(𝑥 ) = 1.5 −
𝑥2
Now, equate the function to zero which will become
5400
0 = 1.5 −
𝑥2
2700
Solving the equation yields 𝑥 = 60 𝑓𝑒𝑒𝑡 and 𝑦 = = 45 𝑓𝑒𝑒𝑡.
60

Thus, to minimize the cost of the fence the side parallel to the barn should
be 60 feet and each of the sides perpendicular to the barn should be 45
feet.
Example 3. Using a 400 feet of fencing materials, a hog raiser wants to
build a sty that is divided into three equal rectangles. What length and
width will maximize the area?

Solution. We start solving the problem by drawing a diagram like the one
shown below.

The area, as a function of the variables is


A = Length x Width
A = 3𝑥 ∙ 𝑦 Equation 1
We also know that the perimeter of the rectangle is
6x + 4y = 400 Equation 2
Dividing both sides of the equation 2 by 2, we will have
3x + 2y = 200.
Solving the equation in terms of variable y yields,
2y = 200 – 3x
y = 100 – 1.5x
Substituting the value of y to equation 1, we will have
A = 3𝑥 ∙ 𝑦
A(x) = 3x(100 – 1.5x)
= 300x – 4.5x2
Now, let us try to determine the domain of the function. Well, you can not
have a negative length of fence so x can’t be negative. And if you build the
ridiculous sty with no width, all 400 feet of fencing would equal 6x. So,
𝑥 ≥ 0 𝑎𝑛𝑑 6𝑥 ≤ 400
200
𝑥≤ 3

Find the critical number of A(x).


A(x) = 300x – 4.5x2
A’(x) = 300 – 9x
0 = 300 – 9x
9x = 300
100
𝑥= 3
100
is the only critical number
3

Evaluate A(x) at the critical number and at the endpoints of the domain.

100 100 100 2 200


A(0) = 0 𝐴( ) = 300 ( ) − 4.5( ) 𝐴( )=0
3 3 3 3

= 5000
The first and third results above should be obvious because they represent
100
corrals with xero length and zero width. Thus, 𝑥 = maximizes the area.
3

Plug the value of x into 𝑦 = 100 − 1.5𝑥 and you get y = 50


100
So, the largest sty is 3 ∙ , or 100 feet long, 50 feet wide, and has an area of
3
5000 square feet.

ACTIVITIES

Directions. Read each of the problem carefully and then solve following the

steps discussed in the module.

1. A fruit cocktail can is to be made with 50 square inches of tin. What are

the dimensions of the can with greatest volume?

2. Right triangle ABC with hypotenuse passing through the point (2, 5) is

placed in the first quadrant with its legs on the x and y axes. What are the

dimensions and area of the smallest such triangle?

3. The sum of two positive numbers is 60. How can we choose them so as to

maximize their product?

WRAP–UP

To summarize what you have learned today, kindly answer/perform


the following questions/instructions:

1. Briefly describe Optimization Problems.


2. How do we solve optimization problems? Enumerate the steps in
solving optimization problems.

VALUING
There are things in our lives that we wanted to maximize such as our
potential, time and resources. However, there are other things as well that
we would rather minimize than maximize such as our debts, problems and
other stressors. Just like the processes that we undertake in solving
optimization problems, we can actually achieve things that we want to
maximize and minimize in our personal lives by similarly undertaking
processes that require patience and time. Thus, if ever things don’t happen
the way you wanted it, don’t stop and start maximizing whatever you have
to reach that goal.

POSTTEST

Directions. Read the problem carefully and then answer the given questions.

PROBLEM 1. A rectangular animal enclosure is to be constructed having


one side along an existing long wall and the other three sides fenced. There
is a 100 meter of fence available.
1. If the length and the width of the enclosure are x and y respectively,
define the formula for the Area of the enclosure in terms of y
.

2. What is the largest possible area for the enclosure?


Education, 2010
Thomas, G.B et al. Thomas’ Calculus. United States of America: Pearson
States of America: McGraw Hill Company, 1988
Mendelson, E. 3000 Solved Problems in Calculus. New York, United
Leithold, L.TC7 Leithold. Singapore: Addison Wesley Longman Inc, 1996
Quezon City, Philippines: Commission on Higher Education, 2016
Balmaceda J.M. et al.Teaching Guide for Senior High School: Basic Calculus.
Publishing House, 2004.
Alferez, G.S. Introduction to Calculus. Quezon City, Philippines: MSA
BOOKS
REFERENCES
ACTIVITY POSTEST
1. The can will be 1. 𝐴 = 100𝑦 − 2𝑦 2
about 3 ¼ inches 2. 𝐴 = 1250𝑚2
wide and about 3 RECAP
1. The function has an
¼ inches tall. The
volume of the can absolute maximum value of
is about 27.14 4 at x = -2 and an absolute
cubic inches. minimum value of 0 at x =
2. The base and 0.
height of triangle 2. The function’s absolute
ABC with smallest extrema therefore occur at
area are 4 and 10 the endpoints, g(-2) = -32 PRETEST
units respectively (absolute minimum), and
and thus the area g(1) = 7(absolute 1. (100 -2x)
is 20 square units. maximum). feet
3. The numbers 3. The function’s absolute 2. A(x) = 100x
are 30 and 30. 3
maximum value is ξ9 ≈ – 2x2
2.08, and it occurs at the 3. Width = 25
right endpoint x = 3. The feet Length
absolute minimum value is = 50 feet
0, and it occurs at the
KEY TO CORRECTION
Basic Calculus SENIOR
HIGH
SCHOOL

Module

16
Quarter 3
THE CHAIN RULE OF
DIFFERENTIATION
EXPECTATIONS

Lesson: The Chain Rule of Differentiation

Learning Objectives:

At the end of the learning episode, you are expected to:


1. illustrate the chain rule of differentiation and
2. solve problems using chain rule

PRETEST

Directions. Find the derivatives of the following functions:

1. 𝑦 = (2𝑥 + 1)5

2. 𝑦 = √3𝑥 2 − 4𝑥 + 6
3. 𝑦 = 𝑠𝑒𝑐(𝑡𝑎𝑛 𝑥)
4. 𝑦 = 𝑡𝑎𝑛3 𝑥
1
5. 𝑦 = 𝑥 2 sec (𝑥)

RECAP

Directions: Find two positive numbers that satisfy the given requirements.

1. The sum is P and the product is a maximum.

2. The product is 64 and the sum is a minimum.

3. The product is 147 and the sum of the first number plus three times the
second number is a minimum.
LESSON

Although we can already differentiate √𝑥 and 𝑥 3 + 1, we cannot yet


differentiate √𝑥 3 + 1. For us to be able to do this, we need a rule that tells us
how to differentiate composite functions whose derivatives we already know.
The rule is known as the Chain Rule and is by far the trickiest derivative
rule, but it is not that difficult if you carefully focus on a few important
points.

The Chain Rule


If y is a function of u, and u is a function of x, then
𝑑𝑦 𝑑𝑦 𝑑𝑢
= ∙ 𝑑𝑥 .
𝑑𝑥 𝑑𝑢

EXAMPLE 1
The function 𝑦 = (5𝑥 2 + 1)2 is obtained by composing the functions 𝑦 =
𝑓 (𝑢) = 𝑢2 and 𝑢 = 𝑔(𝑥 ) = 5𝑥 2 + 1. Calculating derivatives, we see that
𝑑𝑦 𝑑𝑢
∙ 𝑑𝑥 = 2𝑢 ∙ 10𝑥
𝑑𝑢

= 2(5𝑥 2 + 1) ∙ 10𝑥
= 100𝑥 3 + 20𝑥

Calculating the derivative from the expanded formula (5𝑥 2 + 1)2 = 25𝑥 4 +
10𝑥 2 + 1 gives the same result:
𝑑𝑦 𝑑
= 𝑑𝑥 (25𝑥 4 + 10𝑥 2 + 1) = 100𝑥 3 + 20𝑥
𝑑𝑥

EXAMPLE 2 Find the derivative of 𝑦 = √𝑥 3 + 1


Solution. Here 𝑦 = 𝑓(𝑔(𝑥 )), where 𝑓 (𝑢) = √𝑢 and 𝑔(𝑥 ) = 𝑥 3 + 1 . Since the
derivatives of f and g are
1
𝑓′(𝑢) = and 𝑔′(𝑥) = 3𝑥
2√𝑢

We see that
𝑑𝑦 𝑑𝑢 1
∙ = ∙ 3𝑥
𝑑𝑢 𝑑𝑥 2√𝑢

1
= ∙ 3𝑥
2√𝑥 3 + 1
3𝑥
=
2√𝑥 3 +1

3𝑥√𝑥 3 +1
= 2𝑥 3 +2

EXAMPLE 3 Find the derivative of 𝑦 = (8𝑥 − 3)22


Solution. Here, the outside is the 22nd power; it must be differentiated first
and the result multiplied by the derivative of the expression 8x – 3:
𝑑
(8𝑥 − 3)22 = 22(8𝑥 − 3)21 (8)
𝑑𝑥
= 176(8𝑥 − 3)21

EXAMPLE 4 Find the derivative of 𝑦 = sin (𝑥 2 )


Solution. Let 𝑦 = sin (𝑢) 𝑎𝑛𝑑 𝑢 = 𝑥 2
𝑑𝑦 𝑑𝑢
Then = 𝑐𝑜𝑠(𝑢) 𝑎𝑛𝑑 = 2𝑥
𝑑𝑢 𝑑𝑥
𝑑𝑦 𝑑𝑦 𝑑𝑢
Thus = ∙ = 𝑐𝑜𝑠(𝑢) ∙ 2𝑥
𝑑𝑥 𝑑𝑢 𝑑𝑥

= 2𝑥 𝑐𝑜𝑠(𝑥 2 )

EXAMPLE 5 Find the derivative of 𝑦 = tan (𝑥 3 + 1)


Solution. Let 𝑦 = tan(𝑢) 𝑎𝑛𝑑 𝑢 = 𝑥 3 + 1
𝑑𝑦 𝑑𝑢
Then = 𝑠𝑒𝑐 2(𝑢) 𝑎𝑛𝑑 = 3𝑥 2
𝑑𝑢 𝑑𝑥
𝑑𝑦 𝑑𝑦 𝑑𝑢
Thus = ∙ = 𝑠𝑒𝑐 2 (𝑢) ∙ 3𝑥 2
𝑑𝑥 𝑑𝑢 𝑑𝑥

= 3𝑥 2 𝑠𝑒𝑐 2(𝑥 3 + 1)

EXAMPLE 6 Find the derivative of 𝑦 = √𝑠𝑖𝑛(𝑥 3 + 1)


Let 𝑦 = √𝑢 and 𝑢 = sin(𝑝) 𝑎𝑛𝑑 𝑝 = 𝑥 3 + 1
𝑑𝑦 1 𝑑𝑢 𝑑𝑝
Then = = cos(𝑝) = 3𝑥 2
𝑑𝑢 2√𝑢 𝑑𝑝 𝑑𝑥
𝑑𝑦 𝑑𝑦 𝑑𝑢 𝑑𝑝
We know that = 𝑑𝑢 ∙ ∙ .
𝑑𝑥 𝑑𝑝 𝑑𝑥
𝑑𝑦 1
Thus = ∙ cos(𝑝) ∙ 3𝑥 2
𝑑𝑥 2√𝑢
1
= ∙ cos(𝑥 3 + 1 ) ∙ 3𝑥 2
2√sin(𝑥 3 +1 )

3𝑥 2 ∙cos(𝑥 3 +1 )
=
2√sin(𝑥 3 +1 )

EXAMPLE 7 Evaluate the derivatives of the following functions:


a. sin(𝜋𝑥 ) + cos (2𝑥) b. 𝑥 3 𝑠𝑖𝑛√𝑥
Solution.
a. sin(𝜋𝑥 ) + cos (2𝑥)
By the Sum Rule and the Chain Rule:
𝑑
( 𝑠𝑖𝑛𝜋𝑥 + cos(2𝑥 )) = cos (𝜋𝑥)𝜋 – 𝑠𝑖𝑛(3𝑥) ∙ 3
𝑑𝑥

= 𝜋 cos (𝜋𝑥) – 3𝑠𝑖𝑛(3𝑥)


b. 𝑥 3 𝑠𝑖𝑛√𝑥
By the Product Rule and the Chain Rule:
𝑑 3 1
𝑥 𝑠𝑖𝑛√𝑥 = 3𝑥 2 𝑠𝑖𝑛√𝑥 + 𝑥 3 (𝑐𝑜𝑠√𝑥)
𝑑𝑥 2√𝑥
1 5
= 3𝑥 2 𝑠𝑖𝑛√𝑥 + 𝑥 2 (𝑐𝑜𝑠√𝑥)
2
EXAMPLE 8 An object moves along the x- axis so that its position at any
time t ≥ 0 is given by 𝑥 (𝑡) = cos (𝑡 3 + 1). Find the velocity of the object as a
function of t.
𝑑𝑥
Solution. We know that the velocity is . In the instance, x is a
𝑑𝑡

composition of two functions: 𝑥 = 𝑐𝑜𝑠(𝑢) and 𝑢 = 𝑡 3 + 1 . We have


𝑑𝑥 𝑑𝑢
= −sin (𝑢) and = 3𝑡 2
𝑑𝑢 𝑑𝑡

By the Chain Rule,


𝑑𝑥 𝑑𝑥 𝑑𝑢
= ∙
𝑑𝑢 𝑑𝑢 𝑑𝑡
= − sin(𝑢) ∙ 3𝑡 2
= − sin(𝑡 3 + 1) ∙ 3𝑡 2
= −3𝑡 2 sin(𝑡 3 + 1)
ACTIVITIES

A. Directions. Decompose the following functions and complete the table.

𝑦 = 𝑓(𝑔(𝑥 )) 𝑢 = 𝑔(𝑥) 𝑦 = 𝑓(𝑢)

1. 𝑦 = (4𝑥 − 7)3

1
2. 𝑦 =
√2𝑥 + 3

3. 𝑦 = 𝑠𝑖𝑛3 𝑥

4. 𝑦 = 4tan (𝜋𝑥 3 )

5. 𝑦 = (𝑙𝑛𝑥)4

B. Directions. Find the derivatives of the following functions.

1. 𝑦 = (4𝑥 − 5)6
3
2. 𝑦 = √6𝑥 2 + 1

3. 𝑦 = sin (𝑥 4 − 𝑥 3 )

WRAP–UP

To summarize what you have learned today, kindly answer/perform


the following questions/instructions:

1. Briefly describe what is Chain Rule all about.


2. Briefly describe how do we find the derivative of a function applying
chain rule.
VALUING

The chain rule is probably the trickiest among any other rules in
differentiation. However, applying chain rule makes it possible for us to find
the derivative of a composite function with ease and less worried. The chain
rule teaches us that no matter how big your problems are, there are
certainly ways on how you will deal with it in such a way that you do not
feel the burden of solving it. As other people say, “Problems come and go.”,
so do not bother yourself thinking about them, instead find ways on how
you will address them without stressing yourself.

POSTTEST

Directions. Find the derivatives of the following functions:

1. 𝑦 = (4𝑥 − 1)3

2. 𝑦 = √5 − 𝑡
1
3. 𝑦 = 𝑥−2

4. 𝑦 = 𝑐𝑜𝑠 4𝑥
𝑐𝑜𝑡𝑥
5. 𝑦 = 𝑠𝑖𝑛𝑥

KEY TO CORRECTION

5. 2𝑥𝑠𝑒𝑐 𝑥 − sec ቀ𝑥ቁ tan (𝑥) 𝑠𝑖𝑛 3 𝑥


5.
1 1 1 −1−𝑐𝑜𝑠 2𝑥
3. 7 𝑎𝑛𝑑 21
4. 3𝑡𝑎𝑛2 𝑥𝑠𝑒𝑐 2 𝑥 4. −4𝑠𝑖𝑛4𝑥
2. 8 𝑎𝑛𝑑 8
(sec(𝑡𝑎𝑛𝑥 ) tan(𝑡𝑎𝑛𝑥))𝑠𝑒𝑐 2 𝑥 2 2
1. 𝑎𝑛𝑑 (𝑥−2)2
𝑃 𝑃 3.
3. −1
RECAP
2√5−𝑡
√3𝑥 2 −4𝑥+6 2.
2. −1
3𝑥−2
1. 12(4𝑥 − 1)2
1. 10(2𝑥 + 1)4
POSTTEST
PRETEST
REFERENCES

BOOKS

Alferez, G.S. Introduction to Calculus. Quezon City, Philippines: MSA


Publishing House, 2004.

Balmaceda J.M. et al.Teaching Guide for Senior High School: Basic Calculus.
Quezon City, Philippines: Commission on Higher Education, 2016
Leithold, L.TC7 Leithold. Singapore: Addison Wesley Longman Inc, 1996

Mendelson, E. 3000 Solved Problems in Calculus. New York, United


States of America: McGraw Hill Company, 1988

Thomas, G.B et al. Thomas’ Calculus. United States of America: Pearson


Education, 2010
Basic Calculus SENIOR
HIGH
SCHOOL

Module

17
Quarter 3
IMPLICIT
DIFFERENTIATION
EXPECTATIONS

Lesson: Implicit Differentiation

Learning Objectives:

At the end of the learning episode, you are expected to:


1. illustrate implicit differentiation and
2. solve problems using implicit differentiation.

PRETEST

𝑑𝑦
Directions. Use implicit differentiation to find .
𝑑𝑥

1. 𝑥 2 𝑦 + 𝑥𝑦 2 = 7

2. 𝑥 3 − 𝑥𝑦 + 𝑦 3 = 4

3. 𝑥 = 𝑡𝑎𝑛𝑦

4. 𝑥 + tan(𝑥𝑦) = 0

5. 𝑥 4 − 𝑠𝑖𝑛𝑦 = 𝑥 3 𝑦 2

RECAP

𝑑𝑦
Directions: Write the function in the form 𝑦 = 𝑓 (𝑢)𝑎𝑛𝑑 𝑢 = 𝑔(𝑥 ). Then find 𝑑𝑥 as
a function of.

1. 𝑦 = (3𝑥 + 2)6

2. 𝑦 = 𝑡𝑎𝑛3 𝑥
LESSON

Not all functions can be easily written in a form where the independent
variable is completely isolated from the dependent variable, and for some
relations it is simply not possible. Functions and relations of these types are
called implicit. Equations like
𝑥 3 + 𝑦 3 + 4𝑥𝑦 = 0, 𝑦 2 − 2𝑥 = 0, or 𝑥 2 + 𝑦 2 − 36 = 0
define an implicit relation between the variables x and y, meaning that a value
x determines one or more values of y, even though we do not have a simple
formula for the y values.
In our previous examples on differentiation, we have dealt with
equations in the form 𝑦 = 𝑓(𝑥) that expresses y explicitly in terms of the
variable x. But when you are unable to solve for y as a function of x, like the
example
𝑥 2 − 3𝑦 3 + 5𝑦 = 7
the rules on differentiation that we have learned in the previous lessons may
𝑑𝑦
not work anymore. To find for the given equation, we will use Implicit
𝑑𝑥

Differentiation

STEPS IN PERFORMING IMPLICIT DIFFERENTIATION


1. Differentiate both sides of the equation with respect to x.
𝑑𝑦
2. Collect all terms involving on the left side of the equation and
𝑑𝑥
move all other terms to the right side of the equation.
𝑑𝑦
3. Factor out of the left side of the equation.
𝑑𝑥
𝑑𝑦
4. Solve for .
𝑑𝑥
EXAMPLE 1
𝑑𝑦
Find 𝑑𝑥
given that 𝑦 3 − 𝑥 2 = 𝑥 + 𝑦.

Solution.
1. Differentiate both sides of the equation with respect to x.
𝑑 𝑑
(𝑦 3 − 𝑥 2 ) = (𝑥 + 𝑦)
𝑑𝑥 𝑑𝑥
𝑑 𝑑 2 𝑑 𝑑
(𝑦 3 ) − (𝑥 ) = (𝑥) + ( 𝑦)
𝑑𝑥 𝑑𝑥 𝑑𝑥 𝑑𝑥
𝑑𝑦 𝑑𝑦
3𝑦 2 − 2𝑥 = 1 +
𝑑𝑥 𝑑𝑥
𝑑𝑦
2. Collect all terms involving on the left side of the equation and move all
𝑑𝑥
other terms to the right side of the equation.
𝑑𝑦 𝑑𝑦
3𝑦 2 − = 2𝑥 + 1
𝑑𝑥 𝑑𝑥
𝑑𝑦
3. Factor 𝑑𝑥
out of the left side of the equation.

𝑑𝑦
(3𝑦 2 − 1) = 2𝑥 + 1
𝑑𝑥
𝑑𝑦
4. Solve for .
𝑑𝑥

𝑑𝑦 2𝑥 + 1
= 2
𝑑𝑥 3𝑦 − 1
EXAMPLE 2
𝑑𝑦
Find given that 𝑥 2 𝑦 − 𝑥𝑦 2 + 𝑦 2 = 0
𝑑𝑥

Solution.
𝑑 2 𝑑 𝑑 2
(𝑥 𝑦) − (𝑥𝑦 2 ) + (𝑦 ) = 0
𝑑𝑥 𝑑𝑥 𝑑𝑥
𝑑𝑦 𝑑𝑦 𝑑𝑦
𝑥2 + 2𝑥𝑦 − [ (𝑥 ∙ 2𝑦) + 𝑦 2 ] + 2𝑦 =0
𝑑𝑥 𝑑𝑥 𝑑𝑥
𝑑𝑦 𝑑𝑦 𝑑𝑦
𝑥2 + 2𝑥𝑦 − 2𝑥𝑦 − 𝑦 2 + 2𝑦 =0
𝑑𝑥 𝑑𝑥 𝑑𝑥
𝑑𝑦 𝑑𝑦 𝑑𝑦
𝑥2 − 2𝑥𝑦 + 2𝑦 = 𝑦 2 − 2𝑥𝑦
𝑑𝑥 𝑑𝑥 𝑑𝑥
𝑑𝑦 2
(𝑥 − 2𝑥𝑦 + 2𝑦) = −𝑦 2 − 2𝑥𝑦
𝑑𝑥
𝑑𝑦 𝑦 2 − 2𝑥𝑦
= 2
𝑑𝑥 𝑥 − 2𝑥𝑦 + 2𝑦
EXAMPLE 3
𝑑𝑦
Find 𝑑𝑥
given that 𝑦 𝑠𝑖𝑛𝑥 = 𝑥 3 + 𝑐𝑜𝑠𝑦 .

Solution.
𝑑 𝑑 𝑑
(𝑦 𝑠𝑖𝑛𝑥) = (𝑥 3 ) + (𝑐𝑜𝑠𝑦) Use the Product Rule on the left
𝑑𝑥 𝑑𝑥 𝑑𝑥
𝑑𝑦 𝑑𝑦
(𝑠𝑖𝑛𝑥 ) 2
+ 𝑦𝑐𝑜𝑠𝑥 = 3𝑥 − (𝑠𝑖𝑛𝑦) 𝑑𝑥 side.
𝑑𝑥

𝑑𝑦 𝑑𝑦
(𝑠𝑖𝑛𝑥 ) + (𝑠𝑖𝑛𝑦) = 3𝑥 2 − 𝑦𝑐𝑜𝑠𝑥
𝑑𝑥 𝑑𝑥
𝑑𝑦
(𝑠𝑖𝑛𝑥 + 𝑠𝑖𝑛𝑦) = 3𝑥 2 − 𝑦𝑐𝑜𝑠𝑥
𝑑𝑥
𝑑𝑦 3𝑥 2 − 𝑦𝑐𝑜𝑠𝑥
=
𝑑𝑥 (𝑠𝑖𝑛𝑥 + 𝑠𝑖𝑛𝑦)

EXAMPLE 4
𝑑𝑦
Find 𝑑𝑥
given that 3𝑦 + 𝑙𝑛𝑦 = 4𝑒 𝑥 .

Solution.
𝑑 𝑑 𝑑
(3𝑦) + (𝑙𝑛𝑦) = (4𝑒 𝑥 )
𝑑𝑥 𝑑𝑥 𝑑𝑥
𝑑𝑦 1 𝑑𝑦
3 +( ) = 4𝑒 𝑥
𝑑𝑥 𝑦 𝑑𝑥
𝑑𝑦 1
(3 + ) = 4𝑒 𝑥
𝑑𝑥 𝑦
𝑑𝑦 4𝑒 𝑥 4𝑦𝑒 𝑥
= =
𝑑𝑥 1 3𝑦 + 1
3+𝑦

EXAMPLE 5.
Find the equation of the line tangent
𝑥 3 + 𝑦 2 = 5 at the point (1, 2).

Solution.
𝑑 3 𝑑 𝑑
(𝑥 ) + (𝑦 2 ) = (5)
𝑑𝑥 𝑑𝑥 𝑑𝑥
𝑑𝑦
3𝑥 2 + 2𝑦 =0 Figure 1
𝑑𝑥
𝑑𝑦
3(1)2 + 2(2) 𝑑𝑥 = 0 Substitute (1, 2)

𝑑𝑦 −3
=
𝑑𝑥 4
Using the point – slope formula 𝑦 − 𝑦1 = 𝑚 (𝑥 − 𝑥1 ), we now have the
equation of the tangent line
−3
𝑦− 2= (𝑥 − 1)
4

Example 6
Find the derivative of 𝑦 = 𝑎𝑟𝑐𝑐𝑜𝑠𝑥 using implicit differentiation.
Solution.
Rewrite the function as 𝑐𝑜𝑠𝑦 = 𝑥, then differentiate both sides of the
function.
𝑑 𝑑
(𝑐𝑜𝑠𝑦) = (𝑥)
𝑑𝑥 𝑑𝑥
𝑑𝑦
−𝑠𝑖𝑛𝑦 𝑑𝑥
=1
𝑑𝑦 1
= − 𝑠𝑖𝑛𝑦 = −𝑐𝑠𝑐𝑦.
𝑑𝑥
Figure 2

To write the answer in terms of x, we will go back to 𝑐𝑜𝑠𝑦 = 𝑥. We know that


𝑎𝑑𝑗𝑎𝑐𝑒𝑛𝑡 𝑥
𝑐𝑜𝑠 𝑦 = ℎ𝑦𝑝𝑜𝑡𝑒𝑛𝑢𝑠𝑒 = 1, which means that the opposite side to the angle is
√1 − 𝑥 2 .
ℎ𝑦𝑝𝑜𝑡𝑒𝑛𝑢𝑠𝑒 1
𝑐𝑠𝑐𝑦 = = .
𝑜𝑝𝑝𝑜𝑠𝑖𝑡𝑒 √1−𝑥 2

Thus,
𝑑𝑦 1
= −𝑐𝑠𝑐𝑦 = − .
𝑑𝑥 √1−𝑥 2
ACTIVITIES

𝑑𝑦
Directions. Use implicit differentiation to find .
𝑑𝑥

1. 𝑥 2 − 𝑦 2 = 8
𝑥𝑠𝑒𝑐 2 𝑦 𝑑𝑥
= 5.
𝑐𝑜𝑠𝑥−𝑡𝑎𝑛𝑦−1 𝑑𝑦

4. 𝑑𝑥 = −𝑡𝑎𝑛2 𝑦
2. 𝑥 2 𝑦 + 𝑦 2 𝑥 = −2 𝑑𝑦

𝑥𝑒 𝑦+3 𝑑𝑥
= 3.
3. 𝑦 = sin 𝑥𝑦 10− 𝑒 𝑦 𝑑𝑦

3𝑥 3 𝑦 2 −1 𝑑𝑥
= 2.
1
4. 𝑥 = 𝑠𝑒𝑐 𝑦
1−3𝑥 2𝑦 3 𝑑𝑦

𝑦 𝑑𝑥
=− 1.
𝑥 𝑑𝑦

5. 𝑠𝑖𝑛𝑥 + 2𝑐𝑜𝑠2𝑦 = 1 POSTTEST

WRAP–UP

To summarize what you have learned today, kindly answer/perform


the following questions/instructions:

1. Describe briefly what is implicit differentiation.


2. Enumerate the steps in performing implicit differentiation.

VALUING

At first, one would think that finding the derivative of 𝑥 3 𝑦 + 2𝑥𝑦 2 + 7 =


12 would be a very difficult task to do knowing that it would be tedious or
impossible to isolate x from y in the given equation. But here comes the
implicit differentiation, telling us that we can still find the derivative of a
function y with respect to x without having to solve the given equation for y.
If we are to think the process of finding the derivative of a function as finding
our purpose in life, then implicit differentiation shows us that even if we have
limitations and weaknesses as individuals, it is still possible for us to achieve
greater heights and find the purpose of our existence especially if we are able
to recognize the implicit and explicit love of our Creator.

POSTTEST

𝑑𝑦
Directions. Use implicit differentiation to find 𝑑𝑥
.

1. 𝑥 2 + 𝑦 2 = 7
𝑥𝑠𝑒𝑐 2 𝑦 𝑑𝑥
= 5.
𝑐𝑜𝑠𝑥−𝑡𝑎𝑛𝑦−1 𝑑𝑦

4. 𝑑𝑥 = −𝑡𝑎𝑛2 𝑦
2. 𝑥 3 𝑦 3 − 𝑦 − 𝑥 = 0 𝑑𝑦

𝑥𝑒 𝑦+3 𝑑𝑥
= 3.
3. 𝑥𝑒 𝑦 − 10𝑥 + 3𝑦 = 0 10− 𝑒 𝑦 𝑑𝑦

3𝑥 3 𝑦 2 −1 𝑑𝑥
= 2.
4. 𝑐𝑜𝑡𝑦 = 𝑥 − 𝑦
1−3𝑥 2𝑦 3 𝑑𝑦

𝑦 𝑑𝑥
=− 1.
5. 𝑠𝑖𝑛𝑥 = 𝑥(1 + 𝑡𝑎𝑛𝑦)
𝑥 𝑑𝑦

POSTTEST
KEY TO CORRECTION
𝑐𝑜𝑠𝑦−2𝑥 3𝑦 𝑑𝑥 4𝑠𝑖𝑛2𝑦 𝑑𝑥
= 5. = 5.
3𝑥 2 𝑦 2 −4𝑥 3 𝑑𝑦 𝑐𝑜𝑠𝑥 𝑑𝑦

𝑥 𝑑𝑥 𝑦 𝑦 𝑑𝑥
= 4. = −𝑦 2 cos ቀ ቁ cot ( ) 4.
−𝑐𝑜𝑠2 (𝑥𝑦)−𝑦 𝑑𝑦 1 1 𝑑𝑦

𝑑𝑥 1−𝑥𝑐𝑜𝑠(𝑥𝑦) 𝑑𝑥
= 𝑐𝑜𝑠 2 𝑦 3. = 3.
𝑑𝑦 𝑦𝑐𝑜𝑠(𝑥𝑦) 𝑑𝑦

3𝑦 2 −𝑥 𝑑𝑥 𝑥(𝑥+2𝑦) 𝑑𝑥
= 2. = 2.
𝑦−3𝑥 2 𝑑𝑦 −𝑦(𝑦+2𝑥) 𝑑𝑦

𝑥 2 +2𝑥𝑦 𝑑𝑥 𝑦 𝑑𝑥
= 1. = 1.
−2𝑥𝑦−𝑦 2 𝑑𝑦 𝑥 𝑑𝑦

PRETEST ACTIVITY

𝑑𝑥
= 3𝑡𝑎𝑛2 𝑠𝑒𝑐 2 𝑥2. 𝑢 = 𝑡𝑎𝑛𝑥 ; 𝑦 = 𝑢3 ;
𝑑𝑦
𝑑𝑥
𝑢6; = 18(3𝑥 + 2)5 1. 𝑢 = 3𝑥 + 2; 𝑦 =
𝑑𝑦

RECAP

REFERENCES

BOOKS

Alferez, G.S. Introduction to Calculus. Quezon City, Philippines: MSA


Publishing House, 2004.

Balmaceda J.M. et al.Teaching Guide for Senior High School: Basic Calculus.
Quezon City, Philippines: Commission on Higher Education, 2016
Leithold, L.TC7 Leithold. Singapore: Addison Wesley Longman Inc, 1996

Mendelson, E. 3000 Solved Problems in Calculus. New York, United


States of America: McGraw Hill Company, 1988

Thomas, G.B et al. Thomas’ Calculus. United States of America: Pearson


Education, 2010
Basic Calculus SENIOR
HIGH
SCHOOL

Module

18
Quarter 3
RELATED RATES
EXPECTATIONS

Lesson: Related Rates

Learning Objectives:

At the end of the learning episode, you are expected to:


1. solve situational problems involving rates.

PRETEST

Directions. Read each problem carefully and provide what is required.


1. The radius r and height h of a right circular cylinder are related to the
cylinder’s Volume (V) by the formula 𝑉 = 𝜋𝑟 2 ℎ.
𝑑𝑉
a. How is related to dh/dt if r is constant?
𝑑𝑡
𝑑𝑉
b. How is related to dr/dt if h is constant?
𝑑𝑡
𝑑𝑉
c. How is related to dr/dt and dh/dt if neither r nor h is constant?
𝑑𝑡

2. All edges of a cube expand at a rate of 6 centimeters per second. How fast
is the volume changing when each edge is
a. 2 centimeters?
b. 10 centimeters?
RECAP

𝑑𝑦
Directions. Use implicit differentiation to find .
𝑑𝑥

1. 𝑥𝑦 − 𝑥 + 2𝑦 = 1

2. 𝑥 2 + 𝑥𝑦 = 𝑦 3

3. 𝑡𝑎𝑛(𝑥 + 𝑦) = 𝑥

LESSON

One of the many important applications of derivatives is Related


Rates. This is essentially the study of how two or more quantities that change
with time are connected and can be linked with an equation in which the
relation of their rates of change may be found by differentiating both sides of
the equation.
Solving problems involving related rates needs a systematic approach.
From determining the relationships between quantities to using implicit
differentiation when we examine the rates of change of all quantities with
respect to time.
Here are some tips that you can use in solving problems involving
related rates.
STEPS IN SOLVING PROBLEMS INVOLVING RELATED RATES

1. Read and understand the problem.


2. Make a carefully labelled diagram.

3. Write down and label constants, variables, rates and what is being sought.

4. Write a function that relates the variables.


5. Differentiate all terms with respect to time.

6. Substitute known quantities.


Example 1. A 20 – foot ladder rests against a vertical
wall (see Figure 1). If the bottom of the ladder is sliding
away from the base of the wall at the rate of 2 ft/sec,
how fast is the top of the ladder moving down the wall
when the bottom of the ladder is 6 feet from the base?
Figure 1

Solution. Let x be the distance of the bottom of the ladder from the base of
the wall and let y be the distance of the top of the ladder from the base of the
wall. Since the bottom of the ladder is moving away from the base of the wall
𝑑𝑥 𝑑𝑦
at the rate of 2 ft/sec, = 2. We have to find when x = 6.
𝑑𝑡 𝑑𝑡

By the Pythagorean Theorem,


𝑥 2 + 𝑦 2 = 202
Substituting x = 6 to the equation we will have,
62 + 𝑦 2 = 202
36 + 𝑦 2 = 400
𝑦 2 = 364
𝑦 = 2√91
Differentiating both sides of the equation 𝑥 2 + 𝑦 2 = 202 with respect to t, gives
us
𝑑𝑥 𝑑𝑦
2𝑥 + 2𝑦 =0
𝑑𝑡 𝑑𝑡
𝑑𝑥
Substitute = 2 , 𝑥 = 6 𝑎𝑛𝑑 𝑦 = 2√91
𝑑𝑡

𝑑𝑦
2(6)(2) + 2(2√91) =0
𝑑𝑡
𝑑𝑦
24 + 4√91 =0
𝑑𝑡
𝑑𝑦 −24
= ≈ −0.63
𝑑𝑡 4√91
𝑑𝑦
Since < 0, we conclude that the top of the ladder is sliding down the wall
𝑑𝑡

at the rate of about 0.63 ft/sec.


Example 2. Air is being pumped into a spherical balloon
at a rate of 4 cubic inches per minute. Find the rate of
change when the radius is 6 inches.
Solution. Figure 2
𝑑𝑟
The change in radius is being examined when 𝑟 = 6 𝑖𝑛𝑐ℎ𝑒𝑠.
𝑑𝑡
𝑑𝑉 𝑖𝑛3
The air being pumped in is a rate of change of volume, so = 4 𝑚𝑖𝑛
𝑑𝑡

To find the rate of change of the radius, you must find an equation thar relates
the radius r to the volume V. Thus, we will use the equation
4 3
𝑉= 𝜋𝑟
3
Differentiating both sides of the equation with respect to t produces
𝑑𝑉 𝑑𝑟
= 4𝜋𝑟 2
𝑑𝑡 𝑑𝑡
Substituting the known quantities after differentiating we have,
𝑑𝑟
4 = 4𝜋62
𝑑𝑡
1 𝑑𝑟
𝑖𝑛/𝑚𝑖𝑛 =
36𝜋 𝑑𝑡
1
Thus, the rate of change of the radius of the balloon is 𝑖𝑛/𝑚𝑖𝑛.
36𝜋

Example 3. A zero-depth pool is angled


downward at 23˚ (see Figure 3). You are
walking steadily toward the deeper water at Figure 3

a rate of 3 feet per second. At the instant you are 14 feet from the edge of the
water, how fast is the water level rising on you?

Solution. Let x be the distance you have walked into the pool and let h be the
depth of the pool.
𝑑𝑥 𝑓𝑡
At the instant you are examining, 𝑥 = 14 𝑓𝑡 and =3 . We are asked to
𝑑𝑡 𝑠𝑒𝑐
𝑑ℎ
find , the rate of change of depth with respect to time.
𝑑𝑡

The variables are related by tan(230 ) = , or x ∙ tan(230 ) = ℎ.
𝑥

We differentiate the given equation with respect to time


x ∙ tan(230 ) = ℎ
dx 𝑑ℎ
∙ tan(230 ) =
𝑑𝑡 𝑑𝑡
𝑑𝑥 𝑑ℎ
Substitute = 3 to solve
𝑑𝑡 𝑑𝑡

𝑑ℎ
3 ∙ tan(230 ) =
𝑑𝑡
𝑑ℎ
= 3 ∙ tan(230 ) ≈ 1.27 𝑓𝑡/𝑠𝑒𝑐
𝑑𝑡
It is completely fine to insert the 𝑡𝑎𝑛230 value into the equation before
differentiating , because the angle of the pool never changed. However, the 14
which was the value of x never really mattered in the problem because the
water depth was changing at a steady rate the whole time.

Example 4. Water runs into a conical tank at the rate of


10 ft3/min. The tank stands point down and has a height of
12 ft and a base radius of 6 ft. How fast is the water level
rising when the water is 8ft deep? Figure 4

Solution. Figure 4 shows a conical tank filled with water at a height of y


and radius x. The variables in the problem are
V = volume(ft3) of the water in the tank at time t
x = radius (ft) of the surface of the water time t
y = depth (ft) of the water in the tank at time t
We assume that V, x and y are differentiable functions of t.
𝑑𝑦
We will solve for when
𝑑𝑡
𝑑𝑉 10 𝑓𝑡3
𝑦 = 8 𝑓𝑡 and = .
𝑑𝑡 𝑚𝑖𝑛

The water forms a cone with a volume. Thus, we will use the equation
1
𝑉 = 3 𝜋𝑥 2 𝑦

The formula involves x as well as V and y. Because no information is given


𝑑𝑥
about x and at the time in question we need to eliminate x. The similar
𝑑𝑡

triangles as seen Figure 2 tells us that we express x in terms of y:


𝑥 6 𝑦
= 12 or 𝑥 =
𝑦 2
Therefore , we find
1 𝑦
𝑉 = 3 𝜋(2 )2 𝑦
𝜋
= 12 𝑦 3

Solving for the derivative


𝑑𝑉 𝜋 𝑑𝑦
= ∙ 3𝑦 2
𝑑𝑡 12 𝑑𝑡
𝑑𝑉 𝜋 2 𝑑𝑦
= 𝑦
𝑑𝑡 4 𝑑𝑡
𝑑𝑉 𝑑𝑦
Substitute y = 8 and = 10 to solve .
𝑑𝑡 𝑑𝑡

𝜋 2 𝑑𝑦
10 = (8 )
4 𝑑𝑡
𝑑𝑦 5
= ≈ 0. 20
𝑑𝑡 8𝜋

Thus, the water level at the tank is rising at about 0.20 ft/min.

ACTIVITIES

𝑑𝑦 𝑑𝑥
A. Directions. Find the required values of 𝑎𝑛𝑑 assuming that x and
𝑑𝑡 𝑑𝑡

y are both differentiable functions of t.


Equation Find Given
𝑑𝑦 𝑑𝑥
1. 𝑦 = 3𝑥 2 − 5𝑥 𝑎. 𝑑𝑡 𝑤ℎ𝑒𝑛 𝑥 = 3 = 32
𝑑𝑡

𝑑𝑥 𝑑𝑦
𝑏. 𝑤ℎ𝑒𝑛 𝑥 = 2 =4
𝑑𝑡 𝑑𝑡
𝑑𝑦 𝑑𝑥
2. 𝑥𝑦 = 4 𝑎. 𝑤ℎ𝑒𝑛 𝑥 = 8 = 10
𝑑𝑡 𝑑𝑡

𝑑𝑥 𝑑𝑦
𝑏. 𝑤ℎ𝑒𝑛 𝑥 = 1 = −6
𝑑𝑡 𝑑𝑡

B. Read and answer the problem carefully.

1. A spherical balloon is inflated with gas at the rate of 800 cubic centimeters
per minute. How fast is the radius of the balloon increasing at the instant the
radius is (a) 30 centimeters and (b) 60 centimeters?
WRAP–UP

To summarize what you have learned today, kindly answer/perform


the following questions/instructions:

1. What are related rates problems? Briefly describe this application


of derivatives in Calculus.
2. Enumerate the steps in solving problems involving related rates.

VALUING

In problems involving related rates, we are interested at determining


how fast one of the quantities is changing when the other quantity is also
changing. We are working on two quantities which are related to each other.
Considering your relationship to our God as one quantity, your path towards
success as the other quantity and the speed of change as getting closer to
any of the two quantities, how will you describe the status of these two
quantities in your life?

POSTTEST

Directions. Read each problem carefully and provide what is required.


1. The radius r and height h of a right circular cone are related to the cone’s
1
Volume V by the formula 𝑉 = 𝜋𝑟 2 ℎ.
3
𝑑𝑉
a. How is related to dh/dt if r is constant?
𝑑𝑡
𝑑𝑉
b. How is related to dr/dt if h is constant?
𝑑𝑡
𝑑𝑉
c. How is related to dr/dt and dh/dt if neither r nor h is constant?
𝑑𝑡
2. All edge of a cube expanding at a rate of 6 centimeters per second. How
fast is the surface area changing when each edge is
a. 2 centimeters?
b. 10 centimeters?

KEY TO CORRECTION
𝑠𝑒𝑐 𝑑𝑡
= 1800 b.
𝑐𝑚3 𝑑𝑉

2. 𝑎. 𝑑𝑡 = 72 𝑠𝑒𝑐 𝑐𝑚/𝑚𝑖𝑛
18𝜋
=
𝑑𝑡
𝑏.
𝑐𝑚3 𝑑𝑉 1 𝑑𝑟
𝑑𝑡 𝑑𝑡 𝑑𝑡
𝑑𝑟
+ 2𝜋𝑟ℎ
𝑑ℎ
= 2𝜋𝑟 2
𝑑𝑉
c. 𝑥 2 +1
=
𝑑𝑥
3. B. 1. 𝑎. 𝑑𝑡 = 9𝜋 𝑐𝑚/𝑚𝑖𝑛
𝑥2 𝑑𝑦 2 𝑑𝑟

𝑏. 𝑑𝑡 = 2𝜋𝑟ℎ 𝑑𝑡 3𝑦 2 −𝑥
=
𝑑𝑥
2. 𝑏. 𝑑𝑡 = 7
𝑑𝑟 𝑑𝑉 2𝑥+𝑦 𝑑𝑦 4 𝑑𝑥
2+𝑥 𝑑𝑥
𝑑𝑡 𝑑𝑡 = 1.
𝜋𝑟 2 = 𝑎. 1. 𝑑𝑡
𝑑ℎ 𝑑𝑉 1−𝑦 𝑑𝑦 = 26 A. 1. 𝑎.
𝑑𝑦

PRETEST RECAP ACTIVITY

REFERENCES

BOOKS

Alferez, G.S. Introduction to Calculus. Quezon City, Philippines: MSA


Publishing House, 2004.

Balmaceda J.M. et al.Teaching Guide for Senior High School: Basic Calculus.
Quezon City, Philippines: Commission on Higher Education, 2016
Leithold, L.TC7 Leithold. Singapore: Addison Wesley Longman Inc, 1996

Mendelson, E. 3000 Solved Problems in Calculus. New York, United


States of America: McGraw Hill Company, 1988

Thomas, G.B et al. Thomas’ Calculus. United States of America: Pearson


Education, 2010

You might also like